Sei sulla pagina 1di 51

P hysi cs | 23.

25

Solved Examples

JEE Main/Boards Example 2: A circuit draws a power of 550 W from a


source of 220 V, 50Hz. The power factor of the circuit is
Example1: A resistance R, inductance L and a capacitor 0.8 and the current lags in phase behind the potential
C all are connected ina series with an AC supply. The difference. To make the power factor of circuit as 1.0,
resistance of R is 16 Ω, and for a given frequency, the what capacitance will be connected in the circuit?
inductive reactance of L is 24 Ω,and capacitive reactance
of C is 12 Ω. If the current in the circuit is 5 A, find Sol: In series LR circuit, the current lags the applied
(a) The potential difference across R, L and C
voltage by angle φ and the power factor of circuit is
(b) The impedance of the circuit R
cos φ = . When capacitor is connected
(c) The voltage of AC supply R 2 + ω2L2
(d) Phase angle in series in the circuit, the impedance of the circuit is

R 2 + ( X C − XL )
2
Z= and the power factor of the
Sol: In a series LCR circuit, the impedance of circuit is R
circuit is cos φ = .
R 2 + ( X C − XL ) where XC and XL are the capacitive
2
Z= R 2 + ( XL − X C ) 
2

 
and inductive reactances respectively. Phase difference
 X − XC  We want to find the value of the capacitor to make the
between voltage and current is φ =tan−1  L . circuit’s power factor 1.0
 R 
 
Potential drop across resistance is IR and that across (A) Find the value resistance and inductive reactance.
reactance is IX. For a LR circuit, current lags behind voltage in phase.
(a) Potential difference across The power in AC circuit is given as
(i) Resistance VR =I × R =5 × 16 =80 V
V 2rms × cos φ  … (i)
( )
(ii) Inductor  VL = I × ωL = 5 × 24 = 120 V P=
Z
...(1)

I (1 / ωC ) =5 × 12 =60 V
(iii) Capacitor VC =×
( 220 )
2
V 2rms × cos φ × 0.8
⇒ Z= = = 70.4 Ω
(b) The impedance of the circuit P 550

 1 
2 R
Power factor cos φ = , so we get value of resistance as
(16 ) + ( 24 − 12)
2 2 2
=
Z R +  ωL − = Z
 ωC 
= 20 Ω R = Z × cos φ = 70.4 × 0.8 = 56.32 Ω

Inductive Reactance is
(c) The voltage of AC supply is given by
E =I × Z =5 × 20 =100 V ω
= L (Z 2 2
− R= ) ( 70.4 ) − (56.32)
2 2

ω=
L 42.2 Ω
(d) Phase angle between voltage & current is
(B) Capacitance needed to be connected in circuit to
 ωL − ( 1 / ωC )   24 − 12  make power factor = 1.0
−1
φ =
tan   tan−1  
 R   16  When the capacitor is connected in the circuit.
Impedance
( )
= tan−1 0.75 = 360 52'
 1  
2
R 2 +   ωL − …(ii)
=
Z   ...(2)
 ωC  
  
2 3 . 2 6 | Alternating Current

and power factor is given by as, U= P × t= mc∆θ= ( TC ) ∆θ ;


cos φ =
R
=t
( TC=
) × ∆θ 2 × 10
= 348 348 sec
secs= �5.8 min
5.8min.
 1  
2
P 0.0575
R 2 +   ωL −  
 ωC  
 
Example 4: A 100 V ac source of frequency 500 Hz
1
When cos φ= 1, ωL= …(iii) is connected to a series LCR circuit with L=8.1 mH,
ωC C = 12.5 µ F and R= 10 Ω . Find the potential different
1 1 across the resistance.
From (iii) we get
= C =
ω ( ωL ) 2πf ( ωL ) Sol: For LRC circuit, total potential difference is
1
= 75 × 10−6 F VR2 + ( VC − VL ) .
2
= V=
( 2 × 3.14 × 50 ) ( )
× 42.2
= 75 µF. Inductive reactance,

Therefore to make a circuit with power factor = 1, 75 µF XL = 2π × 500 × 8.1 × 10−3 = 25.45 Ω
capacitor is to be connected in a series with resistance
and inductor. Capacitive reactance,
106
=
XC = 25.45Ω
2π × 500 × 12.5
Example 3: A 750 Hz, 20 V source is connected to
a resistance of 100 ohm, an inductance of 0.1803 ⇒ XL =
XC
Henry and a capacitance of 10 microfarad all in series.
Calculate the time in which the resistance (thermal This is the condition of resonance. This means that total
capacity 2J/°C) will get heated by 10°C. potential drop occurs across the resistance only.

( )
2
∴ V = VR2 + VL − VC = VR = 100 V
Sol: For an LCR circuit, the average power dissipated as
2
Vrms The total potential difference across resistance is the
heat is =
Pav × R , where Z is the impedance of the same as the applied voltage across circuit.
2
Z
circuit.
Product of power and time equals the heat generated. Example 5: A 0.21 H inductor and a 12 Ω resistor
are connected ina series to a 20 V, 50 Hz ac source.
XL = ωL = 2πfL = 2π × 750 × 0.1803
= 849.2 Ω and Calculate the current in the circuit and the phase angle
between the current and the source voltage.
1 1
=
X C =
ωC 2πfC
Sol: In series LR circuit, the current lags voltage by phase
1
= = 21.2Ω  ωL 
2π × 750 × 10−5 angle φ =tan−1   . And RMS value of the current is
 R 
V
So X = XL − X C = 849.2 − 21.2 = 828 Ω Irms = rms where Z is impedance of the circuit.
Z

(100 ) + ( 828 ) =
2 2
And hence=
Z R2 + X2 = 834Ω Impedance =
Z R 2 + (ωL)2 ;

But as in case of ac,


122 + ( 2 * 3.14 * 50 * 0.21 )
2
R Vrms
P= Vrms Irms cos=
φ Vrms × ×
(12 ) + (65.94 )=
av 2
Z Z = 2
67Ω
2 2
 Vrms   20 
i.e.=
Pav   =
×R  100 0.00575W And
 ×= V 200
 Z   834  Current Irms = =
rms
= 3.28A
Z 67

Phase angle φ
P hysi cs | 23.27

 ωL  −1  65.94 
VR V VL
tan−1   = tan  ;
 R   12  20

tan = (5.495 ) = 78.69 ° l


VS
Example 6: A current of 4 A flows in a coil when
connected to a 12 V dc source. If the same coil is
connected to a 22 V, 50 rad/sec ac source, a current of (a) P.D. across R,L and C
2.4 A flows in the circuit. Determine the inductance of (b) Impedance of circuit
the coil. Also, find the power developed in the circuit if
a 2500 µ F condenser is connected in a series with the (c) Voltage of AC supply and
coil. (d) Phase angle

Sol: For dc supply, the coil is purely resistive; inductance Sol: For the LCR circuit, impedance is
does not come into picture. For AC voltage source,the
reactance of the inductor is non-zero. When a capacitor Z= R 2 + (X C − XL )2 .
is connected in a series in a circuit, the impedance of
The phase angle between voltage and current is given
R + ( XL − X C ) .
2 2
circuit is Z =
 X − XC 
The real power in the circuit is by φ =tan−1  L  .
 R
 
V2
P I2=
= R R. (a) P.D. across each component is found below
Z2
12 VL =I XL =5 × 24 =120 V,
Resistance of the coil, R= = 3Ω VR =5 × 16 =80 V
4 VC =I X C =5 × 12 =60 V
( Reactance of inductor in dc circuit is zero)
(b) Using the formula of Impedance
12
Impedance of coil, Z= = 5Ω ;
2.4 Z= R 2 + ( XL − X C )
2

Now, Z=
2
R 2 + ω2L2 ;
(16 ) + ( 24 − 12)
2 2
Z= =20 Ω
2 2
Z −R 4
or=
L = = 0.08 H
ω 50
(c) Voltage of AC source is
Reactance of the capacitor E = IZ = 5 × 20 =
100 V
1 1 (d) Phase angle is
X C= = = 8Ω
ωL 50 × 2500 × 10−6
−1 ( L
X − XC )  24 − 12 
= Φ tan
= tan−1  
R  16 
∴ When the capacitor is connected in series,

Z = R 2 + ( XL − X C ) = 32 + ( 8 − 4 ) =5 Ω
2 2 ( )
= tan−1 0.75 = 360 87'

Example 8: A coil of resistance 20 Ω and inductance


R 3
Power factor, cos φ = = ; 0.5H is switched to dc 200 V supply. Calculate the rate
Z 5 of increase of current:
( 2.4 )
2 2
Power developed P= Irms =
Z cos φ × 3 =17.28 W. (a) At the instant of closing the switch
(b) After one time constant
Example 7: A resistance R, an inductance L, and (c) Find the steady state current in the circuit
capacitor C are connected in series with an AC supply
where R=16 Ω . Inductive reactance X= L 24 Ω and Sol: The current in the LR circuit attains constant value
capacitive reactance X= 12 Ω . If the current in the over a long period of time. Generally, the current in the
circuit is 5 A, find
C
2 3 . 2 8 | Alternating Current

circuit is given by
constant.
( )
=i i0 1 − e− t/ τ where τ is one time
(c) The quality factor Q of the circuit
(d) The amplitude of the voltage across the inductor at
(a) Current at any time is given by: the resonant frequency.

 − 
Rt Sol: When the LCR circuit is set to resonance, the
=i i0  1 − e L   ...(1) … (i)
resonant frequency is f = .
  1 1
  2π LC
Differentiating above equation w.r.t. t, we get
ω0L 1 L
Quality factor is=
Q = .

Rt R R C
dI  V R  − RtL  V
=dI  V . R  e= L ∴ i0 V   ...(2) … (ii)
(a) Using formula of resonant frequency
=dt  R . L  e= ∴ i0 R  ...(2)
dt  R L   R
dI V 200 The resonant frequency, for the circuit is given by
At =
t 0, dI= V= 200= 400 A / s
At =
t 0, dt= L= 0.5= 400 A / s ω0
dt L 0.5 1 1
=f =
L 2π 2π LC
(b) Current after one time constant τ =
R
1 1
From equation (ii) = = 5033Hz

dI
( )(
2π 10 × 10−3 H 100 × 10−9 F )
= 400
= e−1 147.15 A / s
dt (b) At resonance current is Maximum i.e. I0
(c) For steady state t = ∞
V0 200
So from (i) we get i(∞ ) = i0 = 400 A =
I0 = = 10.0 A
R 20.0Ω

Example 9: What is average and RMS current over (c) The quality factor Q of the circuit is given by
half cycle if instantaneous current is given by i=4
sin ωt + 3cos ωt. ?
=
Q
(
ω0L 2π 5033s
=
−1
)(
10.0 × 10−3 H )
R ( 20.0Ω )
Sol: Reduce the given expression of current in standard
( )
=i i0 sin ωt + φ , where i0 is the maximum current
form
= 15.8
in the circuit.
(d) At resonance, the amplitude of the voltage across
Given i = 4 sin ωt + 3cos ωt. the inductor is
4 3 
t  5sin ( ωt + α )
= 5  sin ωt + cos ω= VL = I0 XL= I0 ω0L
0
5 5 
where cos α =
4 3
and sin α = ;
= ( )(
(10.0A ) 2π 5033 s−1 10.0 × 10−3 H )
5 5 = 3.16 × 103 V
Comparing with
=i i0 sin ωt + φ ( )
Example 2: Consider the circuit shown in figure. The
 5   10  sinusoidal voltage source is V (t) = V0 sinωt . If both
i0 = 5 A ; ⇒ irms =  A ; iavg =   A
 2  π  switches s1 and s2 are closed initially, find the following
quantities, ignoring the transient effect and assuming
that R, L, V0 and w are known:
JEE Advanced/Boards (a) The current I(t)as a function of time
(b) The average power delivered to the circuit
Example 1: A sinusoidal voltage V(t) = (200 V) sin ωt is
applied to a series LCR circuit with L=10.0 mH, C=100 (c) The current as a function of time, a long time after
nF and R=20.0 Ω . Find the following quantities: only S1 is opened
(a) The resonant frequency
(b) The amplitude of current at resonance
P hysi cs | 23.29

V Thus, the current as a function of time is


V0  ωL 
=
I(t) I0 sin(ωt −=
φ) sin  ωt − tan−1 
R 2 + ω2L2  R 
R₀
A C Note that in the limit of vanishing resistance R=0,
B
R φ = π / 2 , and we recover the expected result for a

purely inductive circuit.


(d) The capacitance C if both s1 and s2 are opened for a
long time, with the current and voltage in phase.
(d) If both the switches are opened, then this would be
(e) The impedance of circuit when both s1 and s2 are
opened. a driven RLC circuit, with the phase angle φ given by tan
1
(f) The maximum energy stored in the capacitor during XL − X C ωL −
oscillations. φ= = ωc
R R
(g) The maximum energy stored in the inductor during If the current and voltage are in phase, lthen= φ ,
oscillations.
implying tan φ =0. Let the corresponding angular
(h) The phase difference between the current and the
voltage if the frequency of V (t) is doubled. 1
frequency be ω0 ; we then obtain. ω0L = And the
ω0 c
(i) The frequency at which the inductive reactance XL is 1
capacitance is C =
And the capacitance
equal to half the capacitive reactance X C . ω20L
(e) From (d), we see that both switches are opened; the
Sol: In LCR circuit explained above, when the switches circuit is at resonance with XL = XC. Thus, the impedance
of the circuit becomes
are closed, the current follows path of least resistance
R 2 + ( XL − X C ) = R
2
i.e., L and C are short-circuited. Impedance of series Z=

R 2 + ( X C − XL ) . The energy stored


2
LCR circuit is Z =
(e) The electric energy stored in the capacitor is
1 2
in inductor is UL = LI and that stored in capacitor is = UE =
1 2 1
CVC C ( IX C ) It attains maximum when the
2
2 2 2
1
UC = CVc2 . current in at its maximum I0:
2
2
(a) When both switches s1 and s2 are closed, the current 1 2 2 1  V0  1 V02L
goes through only the generator and the resistor, so =U =CI X C  =
C,max
2 0 C 2  R  ω2 C2 2R 2
the total impedance of the circuit is R and the current 0

V0 Where we have used ω0 = 2


1 / LC.
is =
IR (t) sin ωt
R (g) The maximum energy stored in the inductor is given
(b) The average power is given by: by.
V 20 2 V 2
0 1 2 LV0
2
= P(t) =
IR (t)V(t) sin= ωt = =
is given by.UL,max LI
R 2R 2 0 2R 2
(c) If only S1 is opened, after a long time a current
will pass through the generator, the resistor and the (h) If the frequency of the voltage source is double, i.e.,
inductor. For this RL circuit, the impedance becomes ω = 2ω = 1 / LC , then the phase becomes
0
1 1
=Z =  ωL − 1 / ωC 
R 2 + XL2 R 2 + ω2L2 φ =tan−1  
 R 
 ωL 
And the phase angle φ is φ =tan−1 
 R 
 ( ) (
 2 / LC L − LC / 2C
= tan−1 
) 
R 
 
 
 3 L
= tan−1  
 2π C 
 
φ =tan  
 R 
( ) (
 2 / LC L − LC / 2C
. 3 0−1| Alternating Current
2=3tan
) 
 R 
 
 
 3 L V 10 1
= tan−1   =
IrmsinL 2 = − = ;
 2π C  ωL 2 2π × 50 × 10 100π
 
1 1 3
(i) If the inductive reactance in one-half the capacitive Irmsincircuit = + =
50π 100π 100π
reactance,
1 1 1  Example 4: A series LCR circuit containing a resistance
XL = X ; ⇒ ωL =  ; of 120 Ω has angular frequency 4 × 105 rads–1. At
2 C 2  ωC 
resonance, the voltage across resistance and inductance
1 ω0
Then
= ω = are 60 V and 40 V respectively. Find the value of L and
2LC 2 C. At what frequency does the current lag the voltage
by 45o?
Example 3: Two inductances of 5.0 H and 10.0 H
Sol: At resonance, XL = XC. The phase angle by which
are connected in parallel circuit. Find the equivalent
inductance and RMScurrent in each inductor and in  X − XC
the current lags the voltage is φ =tan−1  L


mains circuit when connected to source of 10 V AC.  R
 
I1 For resistance VR = IrmsR;
5.0 H VR 60
or I=
rms
= = 0.5 A
I
I2 10.0 H R 120
For inductor V= Irms ω0L
 V
L

10 V AC
40 = 0.5 × 4 × 105 × L ⇒ L = 2 × 10−4 H
Sol: When two inductors are connected in parallel, the 1
LL
At resonance, XL = XC i.e. ω0L =
net inductance is L = 1 2 . If V is the RMS value of ω0 C
L1 + L2
1 1 1
applied voltage, then RMScurrent through inductor is =
C = = µF
ω20L
( 4 × 10 ) 32
2
V
5
× 2 × 10−4
I= .
XL
When the current lags behind the voltage by = 45o,
=
Let E E0 sin ωt , then current drawn from supply is, XL − X C
using tan φ = , gives
R
 π  E0  π 1
=I I0 sin  ωt −=  sin  ωt −  (Since current lags ωL −
 2  ω L  2  1  ω2 L 
1= ω C ⇒ R =ωL − =ωL −  o 
π
by ) R ωC  ω 
2  

Where L is equivalent inductance of circuit. ∴ωR = ω2L − ω2oL

= I1 + I2
I =
E0
 π
sin  ωt − 
(
120 ω = 2 × 10−4 ω2 − (4 × 105 )2 )
ωL1  2 VL

Source Voltage
E0  π E  π
= sin  ωt −  + 0 sin  ωt −  VL - VC
ωL1  2  ω L 2  2 

I I I 1 1 15 3
= + = + = = ;
o
⇒ 45
L L1 L2 5 10 50 10 VR
i

10
⇒L= H
3 VC
V 10 1
=
IrmsinL1 =− = ;
ωL1 2π × 50 × 5 50π
P hysi cs | 23.31

On solving the above equation,we get V 10


=I V00 sin ( ω=
t + φ ) 10 sin ( 314 t + π / 2 )
ω = 8 × 105 or ω = −2 × 105 =I Z sin ( ω= t + φ ) 19.3 sin ( 314 t + π / 2 )
Z cos 314t 19.3
∵ Frequency can’t be negative = 0.52
= 0.52 cos 314t
∴ Ignoring negative root we have ω = 8 × 105 Hz

Example 5: An inductor of 20mH, a capacitor 100 µF Example 6: A choke coil is needed to operate an arc
and a resistor 50 Ω are connected in a series across lamp at 160 V (rms) and 50 Hz. The lamp has an effective
a source of e.m.f. V=10 sin (314t). Find the energy resistance of 5 Ω when running at 10 A (RMS). Calculate
dissipated in the circuit in 20 minutes. If resistance is the inductance of the choke coil. If the same arc lamp is
removed from the circuit and the value of inductance is to be operated on 160 V (dc), what additional resistance
doubled, then find the variation of current with time in is required? Compare the power losses in both cases.
the new circuit. L R

Sol: For the LCR circuit, the energy dissipated over a Choke Lamp
long
= time is U ( )
Vrms Irms cos φ t . When resistance is VL VR
removed,the circuit becomes LC circuit, the impedance
and hence current changes.

V = V0 sin t
The circuit is as shown in figure. One time cycle
Sol: Choke coil has large inductance and low internal
2π 2π
T= = = 0.02s. So, we have to calculate the resistance, sothere is no power loss in the choke coil.
ω 314 Hence, when alamp of some resistance is connected
average energy at time t>>T. in series with the coil, the net RMS voltage in circuit

(=
Vrms ) ( Vrms )R + ( Vrms )L .
2 2 2
L R C is When the same lamp

is operated on dc, additional resistance in a series is


required to limit the current in the lamp to 10 A.

Voltage drop across the lamp is
10 sin 314t
( Vrms )R = ( Irms )(R ) = 10 × 5 = 50 V Voltage drop across
Energy dissipated in time t
choke coil is
 I V R
U = ( Vrms Irms cos φ ) t =  0 × 0 ×  t
∴ ( Vrms ) = ( Vrms ) − ( Vrms )R
2 2
 
 2 2 Z L

=∴U
V02R
=

t  I0
V0 

= (160 ) − (50 ) =
2 2
152 V
2Z 2
 Z 
As (=
Vrms ) (irms
= ) XL (irms )( 2πfL ) ;
L
102 × 50 × 20 × 60
=∴U = 864.2 J
2 × 3153.7 ( Vrms )L
∴L =
When resistance is removed,and inductance is doubled, ( 2πf )(irms )
then cos φ = 0 ⇒ φ = π / 2
Substituting the values
Value of impedance is 152
L= = 4.84 × 10−2 H
=
Z'
1
− ωL'
=
1
− 314 × 40 × 10−3 Ω ( )( )( )
2 π 50 10
ωC 314 × 10 −4
When lamp is operated on DC supply with a resistance
=19.3 Ω
R’ in series, then voltage drop across the circuit is
And the current in the circuit is found to be
V i (R + R' ) or 160=10(5+R’);
=
∴ R=' 11Ω
2 3 . 3 2 | Alternating Current

Choke coil has no resistance.Therefore,for ac circuit The capacitive reactance


power loss in choke coil is zero, while in case of dc, the
loss due to additional resistance R’ is 1 1 100
= X = C
=
Ω Ω
ωC −6 π
(=
10 ) (11 )
2 2π × 50 × 100 × 10
P= i2R'
= 1100 W

The inductive reactance


Example 7: A series AC circuit contains an inductor
(20 mH), a capacitor (100 µF ) and resistance (50 Ω). AC XL = ωL = 2π × 50 × 20 × 10 −3 Ω = 2π Ω.
source of 12 V (RMS), 50 Hz is applied across the circuit.
Find the energy dissipated in the circuit in 1000 s. 1
The net reactance is X= − ωL
ωC
Sol: The average power dissipated in series LCR circuit
100
is Pav Vrms Irms cos φ . For time t � T , the energy =
= Ω − 2π=
Ω 25.5 Ω
π
dissipated is U = Pavt.
(50 Ω ) + ( 25.5 Ω ) =
2 2
Thus, Z 2 = 3150 Ω2
The time period of the source is,
From (i), average power
T=1/f=20 ms.
and t = 1000 s � T 7200
=
Pav = 2.3 W
The average power dissipated is 3150
∴ The energy dissipated int = 1000s is
( )( )
2
2
Vrms R RVrms 50Ω 12V
=Pav V= = s 2.3 × 103 J
rms
Z Z Z2 Z2 U = Pav × 1000 =

7200
Pav = ...(i) … (i)
Z2 

JEE Main/Boards

Q.5 What is the relation between peak value and root


Exercise 1
mean square value of alternating e.m.f?

Q.1 The resistance of coil for direct current (dc)is 10 Ω . Q.6 Is there any device which may control the direct
When alternating current (ac) is sent through it; will its current without dissipation of energy?
resistance increase, decrease or remain the same?

Q.7 What is the phase relationship between current


Q.2 Prove that an ideal inductor does not dissipate and voltage in an inductor?
power in an A.C. circuit.

Q.8 Find the reactance of a capacitance C at f Hz.


Q.3 What is impedance? Derive a relation for it in an
A.C. Series LCR circuit. Show it by a vector.
Q.9 Prove that an ideal capacitor connected to an A.C.
source does not dissipate power.
Q.4 An A.C. supply E = E0 sinω t is connected to a series
combination of L, C and R. Calculate the impedance
of the circuit and discuss the phase relation between Q.10 State the principle of an A.C. generator.
voltage and current.
Q.11 How are the energy losses reduced in a
transformer?
P hysi cs | 23.33

Q.12 Discusses the principle, working and use of a (c) Determine the rms potential drops across the three
transformer for long distance transmission of electrical elements of the circuit. Show that the potential drop
energy. across the LC combination is zero at the resonating
frequency.
Q.13(a) What will be instantaneous voltage for A.C.
supply of 220 V and 50 Hz? Q.22 A circuit containing a 80 mH inductor and a 60 µF
capacitor in series is connected to 230 V, 50Hz supply.
(b) In an A.C. circuit, the rms voltage is 100 2 V , find
The resistance of the circuit is negligible. (a) Obtain
the peak value of voltage and its mean value during a
the current amplitude and rms values. (b) Obtain the
positive half cycle.
rms value of potential drops across each element, (c)
What is the average transferred to the inductor? (d)
Q.14 What should be the frequency of alternating 200 What is the average power transferred to the capacitor?
V so as to pass a maximum current of 0.9 A through an (e) What is the total average power absorbed by the
inductance of 1 H? circuit? [‘average’ ‘implies’ averaged over one cycle;].

Q.15 An alternating e.m.f of 100 V (r.m.s), 50 Hz is Q.23 Answer the following questions: (a) in any A.C.
applied across a capacitor of 10 µF and a resistor of 100 circuit, is the applied instantaneous voltage equal to the
W in series.Calculate (a) The reactance of the capacitor; algebraic sum of the instantaneous voltage across the
(b) The current flowing (c) the average power supplied. series element of the circuit? Is the same true for rms
voltage? (b) A capacitor is used in the primary circuit of
Q.16 The effective value of current in a 50 cycle A.C. an inductor coil. (c) A supplied voltage signal consists
circuit 5.0 A. What is the value of current 1/300s after of a super position of a D.C voltage and A.C. voltage
it is zero? of high frequency. The circuit consists of an inductor
and a capacitor in series. Show that the D.C. signal will
appear across C and the A.C. signal across L. (c) An
Q.17 A pure capacitor is connected to an ac source of
applied voltage signal consists of a superposition of a
220 V, 50 Hz, what will be the phase difference between
D.C. voltage and an A.C. Voltage of high frequency. The
the current and applied emf in the circuit?
circuit consists of an inductor and a capacitor in series.
Show that the D.C. signal will appear across C and the
Q.18 A 100 Ω resistance is connected to a 220 V, 50 Hz A.C. signal across L. (e) Why is choke coil needed in the
A.C. supply. use of florescent tubes with A.C. mains? Why can we
(a) What is the rms value of current in the circuit? not use an ordinary resistor instead of the choke coil?

(b) What is the net power consumed over a full cycle?


Q.24 An inductance of negligible resistance, whose
reactance is 22 Ω at 200 Hz is connected to a 220 V, 50
Q.19 A pure inductance of 1 H is connected across hertz power line, what is the value of the inductance
a 110V, 70 Hz source, find (a) reactance (b) current and reactance?
(c) peak value of current.

Q.25 An electric lamp market 220 V D.C. consumes a


Q.20 A series circuit contains a resistor of 10 Ω , a current of 10 A. It is connected to 250 V-50 Hz A.C.
capacitor, an ammeter of negligible resistance. It is main through a choke. Calculate the inductance of the
connected to a source 220V-50 Hz, if the reading of choke required.
an ammeter is 2.0 A, calculate the reactance of the
capacitor.
Q.26 A 2 µF capacitor, 100 Ω resistor and 8H inductor
are connected in series with an A.C. source. What
Q.21 A series LCR circuit connected to a variable should be the frequency of this A.C source, for which
frequency 230V source and L=5.0 H,C=80 µF , R=40 Ω . the current drawn in the circuit is maximum? If the peak
(a) Determine the source frequency which drives the value of e.m.f of the source is 200 V, find for maximum
circuit in resonance. current, (i) The inductive and capacitive reactance of
the circuit; (ii) Total impedance of the circuit; (iii) Peak
(b) Obtain the impedance of the circuit and the value of current in the circuit ; (iv) The phase relation
amplitude of the current at the resonating frequency. between voltages across inductor and resistor; (v) The
2 3 . 3 4 | Alternating Current

phase difference between voltage across inductor and and minimum current that can flow in the circuit is
capacitor. 10V
.S
( )
Q.27 A step-down transformer converts a voltage of 0.1 H 10
2200 V into 220 V in the transmission line. Number
of turns in primary coil is 5000. Efficiency of the
transformer is 90% and its output power is 8 kW. 10
Calculate (i) Number of turns in the secondary coil (ii)
input power.
(A) 2 Amp (B) 3 Amp

Q.28 What will be the effect on inductive reactance XL (C) 1 Amp (D) Nothing can be concluded
and capacitive XC, if frequency of ac source is increased?
Q.4 The ratio of time constant in build-up and decay in
Q.29 The frequency of ac is doubled, what happens to the circuit shown in figure is
(i) Inductive reactance (ii) Capacitive reactance? R

2R L
Exersice 2
Single Correct Choice Type V
(A) 1:1 (B) 3:2 (C) 2:3 (D) 1:3
Q.1 A rectangular loop with a sliding connector of
length 10 cm is situated in uniform magnetic field
Q.5 A current of 2A is increased at a rate of 4 A/s
perpendicular to plane of loop. The magnetic induction
through a coil of inductance 2H. The energy stored in
is 0.1 tesla and resistance of connecter (R) is 1 Ω. The
the inductor per unit time is
sides AB and CD have resistance 2 Ω and 3 Ωrespectively.
Find the current in the connecter during its motion with (A) 2 J/s (B) 1 J/s (C) 16 J/s (D) 4 J/s
constant velocity of 1 meter/sec.
A D Q.6 The current in the given circuit is increased with a
rate a=4 A/s. The charge on the capacitor at an instant
when the current in the circuit is 2 amp will be:
2 3
E=4V
R
B C R=1
1 1 1 1
( A ) 110 A (B ) 220 A ( C ) 55 A (D ) 440 A
L=1H
)
C=3F

Q.2 For L-R circuit, the time constant is equal to (A) (A) 4 µC (B) 5 µC
Twice the ratio of the energy stored in the magnetic (C) 6 µC (D) None of these
field to the rate of dissipation of energy in the resistance.
(B) Ratio of the energy stored in the magnetic field to Q.7 A coil of inductance 5H is joined to a cell of emf 6 V
the rate of dissipation of energy in the resistance. through a resistance 10 Ω at time t=0. The emf across
(C) Half the ratio of the energy stored in the magnetic the coil at time t= 2 s is:
field to the rate of dissipation of energy in the resistance.
(A) 3V (B) 1.5V (C) 0.75V (D) 4.5V
(D) Square of the ratio of the energy stored in the
magnetic field to the rate of dissipation of energy in Q.8 The network shown in the figure is part of a
the resistance. complete circuit. If at a certain instant, the current I is
5A and it is decreasing at a rate of 103As -1 then VB-VA
Q.3 In the adjoining circuit, initially the switch S is open. equals.
The switch‘s’ is closed at t=0. The difference between
P hysi cs | 23.35

1 + 5 mH L S
A B
I 15 V A B

(A) 20 V (B) 15 V (C) 10 (D) 5 V


R1 R2
12v
Q.9 In the previous question, if I is reversed in direction,
then VB-VA equals
(A) 5 V (B) 10 V (C) 15 V (D) 20 V (A) VL=12 V; point A is at the higher potential
(B) VL=12 V; point B is at the higher potential
Q.10 Two resistors of 10 Ω and 20 Ω and an ideal
inductor of 10 H are connected to a 2 V battery as (C) VL=6 V; point A is at the higher potential
shown in figure. The key K is inserted at time t=0. The (D) VL=6 V; point B is at the higher potential
initial (t=0) and final (t>=00) current through battery
are
Q.13 The power factor of the circuit shown in figure is
10 H
1/ 2 . The capacitance of the circuit is equal to

V=2sin(100t)
10 20


2V 10 0.1H
1 1 1 1
(A) A, A (B) A , A C
15 10 10 15
(A) 400 µF (B) 300 µF
2 1 1 2 (C) 500 µF (D) 200 µF
(C) A, A (D) A , A
15 10 15 25
Q.14 In the circuit, as shown in the figure, if the value
of R.M.S current is 2.2 ampere, the power factor of the
Q.11 In the circuit shown, the cell is ideal. The coil has
box is
an inductance of 4H and zero resistance. F is a fuse zero
resistance and will blow when the current through it
reaches 5A. The switch is closed at t=0. The fuse will 100 C
blow
1/ Henry


fuse
L Box

Sw

-1
Vrms=220 volt, -100  s
2V
1 3 1
(A) Just after t=0 (B) After 2 (A) (B) 1 (C) (D)
2 2 2
(C) After 5s (D) After 10s
Q.15 When 100 V DC is applied across a solenoid, a
Q.12 The circuit shown has been operating for a long current of 1 A flows in it. When 100 V AC is applied
time. The instant after the switch in the circuit labeled across the same coil, the current drops to 0.5 A. If the
S is opened, what is the voltage across the inductor VL frequency of the AC source is 50 Hz, the impedance
and which labeled point (A or B) of the inductor is at a and inductance of the solenoid are:
higher potential? Take R1=4.0 Ω , R2=8.0 Ω and L= 2.5 (A) 100 Ω , 0.93 H (B) 200 Ω , 1.0 H
H.
(C) 10 Ω , 0.86 H (D) 200 Ω , 0.55 H
2 3 . 3 6 | Alternating Current

Q.16 An ac current is given by I =+


I0 I1 sin ωt then its Previous Years’ Questions
rms value will be
Q.1 When an AC source of emf e=E0sin (100 t) is
(A) I02 + 0.5112 (B) I02 + 0.5102
connected across a circuit, the phase difference between
(C) 0 (D) I0 / 2 the emf and the current i in the circuit is observed to be
π
ahead, as shown in the figure. If the circuit consists
Q.17 The phase difference between current and voltage 4
in an AC circuit is π / 4 radians. If the frequency of AC possibly only of R-C or R-L or L-C in series, find the
is 50 Hz, then the phase difference is equivalent to the relationship between the two elements:  (2003)
time difference:
e i
(A) 0.78 s (B) 15.7 ms
(C) 0.25 s (D) 2.5 ms t

Q.18 Power factor an L-R series circuit is 0.6 and that of


a C-R series circuit is 0.5. If the element (L, C, and R) of
the two circuits are joined in series, the power factor of (A) R=1 K Ω ,C=10 µF (B) R=1 K Ω ,C=1 µF
this circuit is found to be 1. The ratio of the resistance (C) R=1 K Ω ,L=10H (D) R=1 K Ω ,L=1H
in the L-R circuit to the resistance in the C-R circuit is

(A) 6/5 (B) 5/6 (C) 4 (D) 3 3 Q.2 The current I4 through the resistor and voltage vC
3 3 4 across the capacitor are compared in the two cases.
Which of the following is/are true?  (2011)
Q.19 The effective value of current i=2 sin100 π t+2 sin
(100 π t+300) is: (a) IRA > IBR (B) IRA < IRB

(A) 2A (B) 2 2 + 3 A B
(C) IC > IC (D) IC < IC
A B

(C) 4 (D) None of these


Q.3 The network shown in Figure is part of a complete
Q.20 In a series R-L-C circuit, the frequency of the circuit. If at a certain instant the current (I) is 5A and is
source is half of the resonance frequency. The nature of decreasing at a rate of 103 A/s then VB − VA = .........V
the circuit will be  (1997)

(A) Capacitive i
(B) Inductive A 1 B
15 V 5 mH
(C) Purely resistive
(D) Data insufficient Q.4 An arc lamp requires a direct current of 10 A and 80
V to function. If it is connected to a 220 V (rms), 50 Hz
AC supply, the series inductor needed for it to work is
close to: (2016)
(A) 0.08 H (B) 0.044 H
(C) 0.065 H (D) 80 H
P hysi cs | 23.37

JEE Advanced/Boards

Exercise 1
L R

Q.1 In the given circuit, find the ratio of i1 to i2 where i1


is the initial current (at t=0), i2is steady state (at t=∞)
current through the battery.
E S

6 2 mH Q.7 Two coils, 1 & 2, have a mutual inductance = M and


resistance R each. A current flows in coils 1, which varies
with time as: I1 = kt2, where k is constant ‘t’ is time. Find
4 4 the total charge that has flown through coil 2, between
10V t = 0 and t = T.

Q.8 Find the value of an inductance which should be


L connected in series with a capacitor of 5 F, resistance
Q.2 Find the dimension of the quantity , where
symbols have usual meaning. RCV of 10 Ω and an ac source of 50 Hz so that the power
factor of the circuit is unity.
Q.3 In the circuit shown, initially the switch is in position
1 for a long time. Then the switch is shifted to position Q.9 In an L-R series A.C circuit the potential difference
2 for long time. Find the total heat produced in R2. across an inductance and resistance joined in series
are respectively 12 V and 16 V. Find the total potential
R2
difference across the circuit.
2 L
1 S Q.10 A 50W, 100V lamp is to be connected to an ac
mains of 200V, 50Hz. What capacitance is essential to
E R1 be put in series with lamp.

Q.4 Two resisters of 10 Ω and 20 Ω and an ideal Q.11 In the circuit shown in the figure, the switched
inductor of 10 H are connected to a 2V battery as S1 and S2 are closed at time t=0. After time t = (0.1) In
shown in figure. The key K is shorted at time t=0. Find 2sec, switch S2 is opened. Find the current in the circuit
the initial (t=0) and final (t->∞) current through battery. at time t = (0.2) ln 2sec.
L = 10 H
S1

R = 10 40 S2
20
K 100 V 10

1H
Q.5 An emf of 15 V is applied in a circuit containing
5 H inductance and 10 Ω resistance. Find the ratio of
the current at time t=∞ and t=1 second. Q.12 Find the value of i1and i2

S i1 30
Q.6 In the circuit in shown in figure, switch S is closed i2
at time t=0. Find the charge which passes through the 20
battery in one time constant.
100 V
2 3 . 3 8 | Alternating Current

(i) Immediately after the switch S is closed.


V
(ii) Long time later, with S closed.
(iii) Immediately after switch S is open
(iv) Long time after S is opened.

Q.13 Suppose the emf of the battery in the circuit


O T/4 T/2 3T/4 T
shown varies with time t so the current is given by
i(t) = 3+5t, where i is in amperes & t is in seconds. Take
R=4 Ω , L=6H & find an expression for the battery emf
as a function of time.
Find the amplitude of current in the steady state and
R i(t) obtain the phase difference between the current and
the voltage. Also plot the variation of current for one
cycle on the given graph.
 L

Exercise 2
Q.14 An LCR series circuit with 100 Ω resistance Single Correct Choice Type
is connected to an ac source of 200 V and angular
frequency 300rad/s. When only the capacitance is Q.1 A square coil ABCD is placed in x-y plane with its
removed, the current lags behind the voltage by 600. centre at origin. A long straight wire, passing through
When only the inductance is removed, the current origin, carries a current in negative Z-direction. Current
leads the voltage by 600. Calculate the current and the in this wire increases with time. The induced current in
power dissipated in the LCR circuit. the coil is

Q.15 A box P and a coil Q are connected is series with B


y
C
an ac source of variable frequency. The emf source at
10V. Box P contains a capacitance of 1 µ F in series with
a resistance of 32 Ω . Coil Q has a self-inductance 4.9 
mH and a resistance of 68 Ω series. The frequency
x
adjusted so that the maximum current flows in P and Q.
Find the impedance of P and Q atthis frequency. Also
find the voltage across P and Q respectively.
A D

Q.16 A series LCR circuit containing a resister of 120 (A) Clock wise (B) Anti clockwise
Ω has angularresonance frequency 4 × 105 rad s-1. At
resonance, the voltage across resistance and inductance (C) Zero (D) Alternating
are 60V and 40V respectively. Find the values of L and C.
At what frequency current in the circuit lags the voltage Q.2 An electric current i1 can flow in either direction
by 45o? through loop (1) and induced current i2 in loop (2).
Positive i1 is when current is from ‘a’ to ‘b’ in loop (1)
and positive i2 is when the current is from ‘c’ to ‘d’ in
Q.17 In an LR series circuit, a sinusoidal voltage V=V0 loop
sinωt is applied. It is given that
ω Loop (1)
L=
35mH,R =
11Ω, Vrms =
220V, =
50Hz
2π a b
And π =22 / 7 . Loop (2)
c d
P hysi cs | 23.39

(2) In an experiment, the graph of i2 against time ‘t’ is as Q.4 Two identical inductances carry currents that vary
shown below by Figure which one (s) of the following with time according to linear laws (see in figure). In
graphs couldi2have caused i2 to behave as give above. which of the inductances is the self-inductance emf
i2

greater?
i2
(A) 1 (B) 2
(A) t
(A) 0 t (C) Same (D) Data is insufficient to decide
t

i2 i2 Q.5 L, C and R represents physical quantities inductance,


i2 capacitance and resistance. The combination which has
the dimensions of frequency?
11 RR
(A)
(A) t (B)
(B) t (A)
11
and
and
RR
and (B) and
and
and
(B) t RC
RC LL RC
RC LL

1 C
i i2 (C) (D)
i22 LC L

Q.6 In the circuit shown, X is joined to Y for a long time,


and then X is joined to Z, the total heat produced in R2
(C)
(C) (D)
(B) t t
(C) t t
is:
i1 R2
i2
X Z
Q.3 In an L-R circuit connected to a battery of constant
e.m.f. E, switch S is closed at time t = 0. If e denotes the L Y
magnitude of induced e.m.f.
(C) t across inductor and i the
current in the circuit at anytime t. Then which of the E R1
following graphs shows the variation of e with i?
i1 i2
I

(A) (B)

2
1 t t

t i3 i4

e e
(C) (D)
(A) (B)
t t
t t
LE2 LE2 LE2 LE2R 2
(A) (B) (C) (D)
e e 2R12 2R 22 2R1R 2 2R12
(C) (D)
Q.7 An induction coil stores 32 joules of magnetic
energy and dissipates energy as heat at the rate of 320
t t
watt when a current of 4 amperes is passed through it.
Find the time constant of the circuit when the coil is
joined across a battery.
(A) 0.2s (B) 0.1s (C) 0.3s (D) 0.4s
2 3 . 4 0 | Alternating Current

Q.8 In an L-R decay circuit, the initial current at t=0 is 1. Q.14 The current I, potential difference VL across the
The total charge that has inductor has reduced to one- inductor and potential difference VC across the capacitor
fourth of its initial value is in circuit as shown in the figure are best represented
vectorially as.
(A) LI / R (B) LI / 2R (C) LI / 2R (D) None
VC
VC
Q.9 An inductor coil stores U energy when i current
is passed through it and dissipates energy at the rate (A) I (B)
of P. The time constant of the circuit, when the coil is VL VL I
connected across a battery of zero internal resistance is
4U U 2U 2P
(A) (B) (C) (D) VL
P P P U VL
(C) (D)
I I
Q.10 When a resistance R is connected in series with VC VC
an element A, the electric current is found to be
lagging behind the voltage by angle θ1. When the
same resistance is connected in series with element B, Q.15 In the shown AC circuit in figure, phase difference
current leads voltage by θ2. When R, A, B, are connected between current I1 and I2 is
in series, the current now leads voltage by θ. Assume XC
same AC source in used in all cases. Then:
I1
(A) θ = θ1 − θ2 (B) tan=
θ tan θ2 − tan θ1 
I2
θ1 + θ2
(C) θ = (D) None of these XL R
2

Q.11 The power in ac circuit is given by P=ErmsIrms cos φ . π X X − XC


(A) − tan−1 L (B) tan−1 L
The value of cos φ in series LCR circuit at resonance is: 2 R R
1 1
(A) Zero (B) 1 (C) (D) π X X − XC π
2 2 (C) + tan−1 L (D) tan−1 L +
2 R R 2
Q.12 If I1, I2,I3 and I4 are the respective r.m.s values of the
time varying current as shown in figure the four cases
I.II,III and IV in. Then identify the correct relations. Multiple Correct Choice Type

Q.16 A circuit element is placed in a closed box. At time


i i
(I) I0 (II) I0
O t O t t=0, constant current generator supplying a current of
-I0 -I0 1 amp, is connected across the box. Potential difference
across the box varies according to graph shown in
Figure. The element in the box is:
i i
(A) Resistance of 2 Ω (B) Battery of emf 6V
(III) I0 (IV) I0
O t O
-I0 (C) Inductance of 2H (D) Capacitance of 0.5F

(A) I1 = I2 = I3 = I4 (B) I3 > I1 = I2 > I4



(C) I3 > I4 > I2 = I1 (D) I3 > I2 > I1 > I4 8

2
Q.13 In series LR circuit XL=3R. Now a capacitor with XC=R
is added in series. Ratio of new to old power factor is
t(s)
3
(A) 1 (B) 2 (C) 1 (D)
2
2i
P hysi cs | 23.41

Q.17 For L-R circuit, the time constant is equal to (A) The low resistance of P
(A) Twice the ratio of the energy stored in the magnetic (B) The induced-emf in L
field to the rate of the dissipation of energy in the
(C) The low resistance of L
resistance
(D) The high voltage of the battery B
(B) The ratio of the energy stored in the magnetic field
to the rate of the dissipation of energy in the resistance.
Q.20 Two different coils have a self-inductanceof
L
C 8mH and 2mH. The current in one coil is increased at
a constant rate. The current in the second coil is also
VC
VL increased at the same instant of time. The power given
to the two coils is the same. At that time the current,
 the induced voltage and the energy stored in the first
(C) Half of the ratio of the energy stored in the magnetic coil are I1 V1 and W1 respectively. Corresponding values
field to the rate of the dissipation of energy in the for the second coil at the same instant are I2, V2 and W2
resistance. respectively . Then:
(D) Square of the ratio of the energy stored in the I1 1 I
(A) = (B) 1 = 4
magnetic field to the rate of the dissipation of energy I2 3 I2
in the resistance.
W1 V2 1
(C) = 4 (D) =
Q.18 An inductor L, a resistor R and two identical bulbs W2 V1 4
B1 and B2 are connected to a battery through a switch S
as shown in the figure. The resistance of the coil having Q.21 The symbol L, C, R represents inductance,
inductance L is also R. Which of the following statement capacitance and resistance respectively. Dimension of
gives the correct description of the happening when frequency is given by the combination.
the switch S is closed?
1
L B1 (A) 1/RC (B) R/L (C) (D) C/L
LC
R B2
Q.22 An LR circuit with a battery is connected at t=0.
Which of the following quantities is not zero just after
E S the circuit is closed?
(A) Current in the circuit
(A) The bulb B2 lights up earlier then B1 and finally both
the bulbs shine equally bright. (B) Magnetic field

(B) B1 lights up earlier and finally both the bulbs acquire (C) Power delivered by the battery
brightness. (D) Emf induced in the inductor
(C) B2 lights up earlier and finally B1 shines brighter
than B2. Q.23 The switches in figure (a) and (b) are closed at t=0
(D) B1 and B2 lights up together with equal brightness C R L R
all the time. )

Q.19 In figure, a lamp P is in series with an iron-core


inductor L. When the switch S is closed, the brightness
of the lamp rises relatively slowly to its full brightness E E
than it would to without the inductor. This is due to (a) (b)
(A) The charge on C just after t=0 is EC.
P L
(B) The charge on C long after t=0 is EC.

S
(C) The charge on L just after t=0 is E/R.
(D) The charge on L long after t=0 is EC.
B
2 3 . 4 2 | Alternating Current

Q.24 Two coils A and B have coefficient of mutual Comprehension Type Question
inductance M=2H. The Magnetic flux passing through
coil A changes by 4 Weber in 10 seconds due to the Paragraph 1: A capacitor of capacitance C can be
change in current in B. Then charged (with the help of a resistance R) by a voltage
source V, by closing switch s1 while keeping switch s2
(A) Change in current in B in this time interval is 0.5 A
open. The capacitor can be connected in series with an
(B) The change in current in B in this time interval is 2A inductor ‘L’ by closing switch S2 and opening S1.
(C) The change in current in B in this time interval is 8A V
(D) A change in current of 1A in coil A will produce a
change in flux passing through B by 4 Weber.
R S1
Assertion Reasoning Type
C

(A) Statement-I is true, statement-II is true and


statement-II is correct explaining for statement-I.
S2
(B) Statement-I is true, statement-II is true and L
statement-II is not correct explaining for statement-I
(C) Statement-I is true, statement-II is false.
Q.28 After the capacitor gets fully charged, s1 is opened
(D) Statement-I is false, statement-II is true. and S2 is closed so that the inductor is connected in
series with the capacitor. Then,
Q.25 Statement-I: when resistance of rheostat is (A) At t=0, energy stored in the circuit is purely in the
increased, clockwisecurrent is induced in the ring. form of magnetic energy.
Statement-II: Magnetic flux through the ring is out of
the phase and decreasing. (B) At any time t>0, current in the circuit is in the same
direction.
(C) At t>0, there is no exchange of energy between the
inductor and capacitor.
(D) At any time t>0, instantaneous current in the circuit

is V C
L
Q.26 Statement-I: Peak voltage across the resistance
can be greater than the peak voltage of the source in a
series LCR circuit. Q.29 If the total charge stored in the LC circuit is Q0
then for t>=0

Statement-II: Peak voltage across the inductor can be π t 


greater than the peak voltage of the source in a series =
(A) The charge on the capacitor is Q Q0 cos  + 
2 LC 
LCR circuit.
π t 
Q.27 Statement-I: when a circuit having large inductance =
(B) The charge on the capacitor is Q Q0 cos  + 
is switched off, sparking occurs at the switch. 2 LC 

d2Q
(C) The charge on the capacitor is Q = LC
Statement-II: Emf induced in an inductor is given by dt2
di
|e|
∈ =L (A) Statement-I is true, statement-II is true 1 d2Q
dt (D) The charge on the capacitor is Q = −
and statement-II is correct explanation for statement-I. LC dt2
(B) Statement-I is true, statement-II is true and statement-
II is not the correct explanation for statement-I.
(C) Statement-I is true, statement-II is false.
(D) Statement-I is false, statement-II is true.
P hysi cs | 23.43

Paragraph 2: In a series L-R circuit, connected with a ways as shown in column II. When a current I (steady
sinusoidal ac source, the maximum potential difference state for DC or rms for AC) flows through the circuit, the
across L and R are respectively 3 volts and 4 volts corresponding voltage V1 and V2 (indicated in circuits)
are related as shown in column I.  (2010)
Q.30 At an instant, the potential difference across
Column I Column II
resistor is 2 V. The potential difference in volt, across
the inductor at the same instant will be: (A) I ≠ 0, V1 is (p)
V1 V2
(A) 3 cos30o (B) 3 cos60o Proportional to I
(C) 3 cos45o (D) None of these 6 mH 3 F

Q.31 At the same instant, the magnitude of the potential


difference in volt, across the ac source may be V

(B) I ≠ 0, V2 > V1 (q)


4+3 3
(A) 4 + 3 3 (B) V1 V2
2

3 3 6 mH 3 F
(C) 1 + (D) 2 +
2 2

Previous Years’ Questions (C)=


V1 0,=
V2 V (r)
V1 V2

Q.1 A circuit containing a two position switch S is


shown in Figure. 6 mH 2 F

R3 C 
V

2 2  F 1 R5 (D) I ≠ 0, V2 is (t)
R1 E1 V1 V2
Proportional to I
2 12 V 1k 3 F
1 R2
A E2 B

2 S 3V 2 R4
3 V
L
(s)
10 mH V1 V2

(a) The switch S is in two position 1. Find the potential 3 F


difference VA − VB and the rate production of joule heat
6 mH

in R1.

(b) If Now The switch S is put in position 2 at t=0. Find: V

(i) Steady current in R4 and(ii) The time when current


Paragraph 1 (Q.3 to Q.8)
in R4 is half the steady value. Also calculate the energy
stored in the inductor L at that time. (1991) The capacitor of capacitance C can be charged(with the
help of resistance R) by a voltage source V, by closing
Q.2 Match the Columns switch S1 while keeping switch S2 open. The capacitor
You are given many resistances, capacitors and can be connected in series with an inductor L by closing
inductors. They are connected to a variable DC voltage switch S2 and opening S1.
source (the first two circuits) or in AC voltage source of
50 Hz frequency (the next three circuits) in difference
2 3 . 4 4 | Alternating Current

V Q.6 In the circuit shown, A and B are two cells of same


emf E but different internal resistance r1 and r2 ( r1 >r2)
respectively find the value of R such that the potential
S1
difference across the terminals of cell A is zero a long
R
C
time after the key K is closed (2004)
R
S2 R L
L R
A B
R
R
r1 r2
Q.3 Initially, the capacitor was uncharged. Now switch C
s1 is closed and S2 is kept open. If time constant of this R
circuit is τthen  (2006)
S
(A) After time interval τ, charge on the capacitor is CV/2
(B) After time interval 2τ, charge on the capacitor is CV Q.7 In an L-R series circuit, a sinusoidal voltage V = V0
(1-e-2)
sin ωt is applied. It is given that L=35 mH, R=11 Ω ,
(C) The work done by voltage source will be half of the
heat dissipated when the capacitor is fully charged Vrms 220V, ω=
= / 2π 50Hz and
= π 22 / 7.

(D) After time interval 2τ, charge on the capacitor is CV Find the amplitude of current in the steady state and
(1-e-1) obtain the phase difference between the current and
the voltage. Also plot the variation of current for one
cycle on the given graph.  (2004)
Q.4 After capacitor gets fully charged, S1 is opened and
S2 is closed so that the inductor isconnectedin series V
with the capacitor, then (2006)
(A) At t=0, energy stored in the circuit is purely in the
form of magnetic energy. t
T/4 T/2 3T/4 T
(B) At any time t>0, current in the circuit is in the same
direction.
(C) At t>0, there is no exchange of energy between the
inductor and capacitor. Q.8 What is the maximum energy of the anti-neutrino ?
 (2012)
(D) At any time t>0, instantaneous current in the circuit
(A) Zero
C
may V (B) Much less than 0.8 × 106 eV
L
(C) Nearly 0.8 × 106 eV
Q.5 If the total charge stored in the LC circuit is Q0 then
(D) Much larger than 0.8 × 106 eV
for t ≥ 0 (2006)

π t  Q.9 At time t = 0 terminal A in the circuit shown in the


(A) The charge on the capacitor
= is Q Q0 cos  + 
2 LC  figure is connected to B by a key and an alternating
current = I(t) I0 cos (ωt) , with I0 = 1A and ω =500
π t  rad/s starts flowing in it with the initial direction shown
(B) The charge on the capacitor
= is Q Q0 cos  − 
2 LC  7π
in the figure. At t = , the key is switched from B to
d2Q 6ω
(C) The charge on the capacitor is Q = LC D. Now onwards only A and D are connected. A total
dt2
charge Q flows from the battery to charge the capacitor
1 d2Q fully. If C = 20 µF, R =10Ω and the battery is ideal with
(D) The charge on the capacitor is Q = −
LC dt2 emf of 50 V, identify the correct statement(s). (2014)
P hysi cs | 23.45

B D (A) Magnitude of the maximum charge on the capacitor



A before
= t is 1 × 10−3 C

 50 V (B) The current in the left part of the circuit just before
C=20F

t= is clockwise.

R=10 (C) Immediately after A is connected to D, the current
in R is 10 A.
(D) Q= 2 × 10−3 C

MASTERJEE Essential Questions

JEE Main/Boards JEE Advanced/Boards

Exercise 1 Exercise 1
Q. 15 Q.21 Q.22 Q. 3 Q.4 Q.7

Q.23 Q.27 Q.14 Q.15 Q.16

Exercise 2 Exercise 2
Q. 1 Q.3 Q. 11 Q.2 Q.3 Q.12

Q.12 Q.14 Q.22 Q.23


Q.28 Q.28 Q.29
Q.30 Q.31

Answer Key

JEE Main/Boards Q.10 It is based up on the principle of electromagnetic


induction.
Exercise 1 Q.11 (i) By using laminated iron core, we minimize loss
V  of energy due to eddy current.
Q.5. Vrms =  0 
(ii) By selecting a suitable materials for the core of a
 2
transformer, the hysteresis loss can be minimized.
Q.6 No
Q.13 (a) ≈ 311sin314t (b) 200V, 127.4V
Q.7 The current lags behind the voltage by phase angle
π/2. Q.14 50Hz
1 1 Q.15 (a) 318.31 Ω (b) 0.527 A (c) 9 W
Q.8 Capacitive reactance, =
XC =
ωC 2πfc Q.16 6.124A
2 3 . 4 6 | Alternating Current

Q.18 (a) 2.20A, (b) 484 W Q.23 (a) Yes. The same is not true for rms voltage,
because voltage across different element may not be
Q.19 0.354A in phase.
Q.20 109.5 A (b) The high induced voltage, when the circuit is broken,
Q.21 (a) 50 rad s-1, (b) 40 Ω , 8.1A, (c) VLcms=1437.5 is used to change the capacitor, thus avoiding sparks,
etc.
 1 
V,Vvcrms=1437.5V,VRms=230 VLCrms= Irms  ω0L − =  0 (c) For dc, impedance of L is negligible and C very
 ω0 C 
high (infinite), so the D.C. signal appears across C. For
Q.22 (a) For
= V V0 sin ωt frequency ac, impedance of L is high and that of C is
low. So, the A.C. signal appears across L.
V0  π
=I sin  ω
= t +  ; If R 0 (e) A choke coil reduces voltage across the tube without
1  2
ωL − wasting power. A resister would waste power as heat.
ωC
Q.24 1.75 × 10−2 H; 5.5Ω
Where- sign appears if ωL >I/ ωC , and+sign appears
if ωL <I ωC . Q.25 0.04H
=I0 11.6A,I
= rms 8.24A Q.26 Resonant frequency=39.79 Hz
(b) VLCrms =207V, VCrms =437 V (i) 2000 Ω (ii) 100 Ω (iii) 2A

(c) Whatever be the current I in L, actual voltage leads (iv) 900 (v)1800
current by π / 2 . Therefore, average power consumed Q.27 (i) 500; (ii) 8.9kW
by L is zero.
(d) For C, voltage lags by π / 2 . Again average power
consumed by C is zero.
(e) Total average power absorbed is zero.

Exercise 2
Q.1 B Q.2 A Q.3 C Q.4 B Q.5 C Q.6 C
Q.7 A Q.8 B Q.9 C Q.10 A Q.11 D Q.12 D
Q.13 C Q.14 A Q.15 D Q.16 A Q.17 D Q18 D
Q.19 B Q.20 A

Previous Years’ Questions


Q.1 A Q.2 B, C Q.3 15V Q.4 C

JEE Advanced/Boards
Exercise 1
LE2
Q.1 0.8 Q.2 [I]−1 Q.3
2R12

1 1 e2 − 1 EL
Q.4 , Q.5 Q.6
15A 10A e 2
eR 2

KLt2 20
Q.7 q = C Q.8 ≅ 2H Q.10 C = 9.2 .F
R π2
P hysi cs | 23.47

Q.11 6.94 A Q.12 (i) i1 = i2 = 10/3A, (ii) i1 = 0, i2 = 30/11A, (iv) i1 = i2 = 0

Q.13 42+20t V Q.14 2A, 400W

1
Q.15 Z = 100 Ω, VQ = 9.8 V Q.16 0.2 mH, µF,8 × 105 rad / s
32

π  1
Q.17 20A, , ∴ Steady state current=20sin π  100t − 
4  4

Exercise 2

Single Correct Choice Type

Q.1 C Q.2 D Q.3 A Q.4 A Q.5 A Q.6 A


Q.7 A Q.8 B Q.9 C Q.10 B Q.11 B Q.12 B
Q.13 D Q.14 D Q.15 A

Multiple Correct Choice Type

Q.16 D Q.17 D Q.18 A Q.19 B Q.20 B, C, D Q.21 A, B, C


Q.22 D Q.23 B, D Q.24 D

Assertion Reasoning Type

Q.25 C Q.26 D Q.27 A

Comprehension Type

Paragraph 1: Q.28 D Q.29 C

Paragraph 2: Q.30 D Q.31 B

Previous Years’ Questions


Q.1 (a) -5v, 24.5w (b) (i) 0.6A (ii) 1.386 × 10–3s, 4.5 × 10–4 J

Q.2 A → r, s, t; B → q, r, s, t; C →q, p; D → q, r, s, t

4
Q.3 B Q.4 D Q.5 C Q.6 R = (r – r )
3 1 2
π
Q.7 Amplitude = 20A, phase difference = Q.8 C
4
Q.9 C, D
2 3 . 4 8 | Alternating Current

Solutions

JEE Main/Boards Im

z
Exercise 1
x
z
Sol 1: In a resistance coil, when an alternating current 
is flown, there will be a magnetic field generated across R
Re
the coil and so there will be an inductance induced into
the coil. Hence it will have more impedance compared
Sol 4: As derived above,
to the one withDC current.
ZR = R
Sol 2: We know that power dissipated = VI cosθ. ZL = iωL
R  ZC = –i/ωC
cos θ =   ⇒ power factor
Z znet=ZR + ZL + ZC (Since they all are in series)
now for an ideal inductor, Z = ωL and R = 0
Now we can write any quantity in phasor notation,
∴ cos θ = 0
for V = V0 cos (ωt + θ)
Hence power = VI (0) = 0
we write this quantity in phasor notation as,

Sol 3: Impedance is the effective resistance of an V = |V| ∠q


electric circuit or component to alternating current, ⇒ V = V0∠θ. [θ is the phase angle].
arising from the combined effect of ohmic resistance
This is very helpful for us.
and reactance.
R L C Now for the given potential, V = V0 sin ωt
π
V = V0 cos (ωt – )
2
V = V0 cos t 
∴ V= V ∠ − π  ........(1) … (i)
0
2
i
We got Znet = ZR + ZL + ZC= R + i ωL –
Now let ‘i’ (iota) be the complex number, square root ωL
of –1.  1 
Znet = R + i  ωL − 
Now, Impedance of resistance ‘R’ = R ≡ ZR  ω C
Impedance of Inductor ‘L’ = i ωL ≡ ZL
2
 1 
 −i  now |Znet| = R 2 +  ωL − 
Impedance of capacitor ‘C’ =   ≡ ZC  ωC 
 ωC 
now net Impedance of the circuit (figure (i)) is  1 
 ωL − 
Znet = ZR+ ZC + ZL tan θ =  ω C
 R 
 
i  1   
=R– + iωL = R + i  ωL − 
ωC  ω C
P hysi cs | 23.49

Im Sol 6: No nothing is perfect. It is impossible to make a


perpetual machine.

z Sol 7: Using the notation used in Q.4 and Q.5;

 L
Re
R

With this we can write V = V0 cos t 


Znet | Znet | ∠θ 
= … (ii)

Now we known that


∴ In phasor notation: V0 = V0∠ 0
V = I × Z [ V = I R]
π
v0∠ −
π ZL = iωL ⇔ ZL = |ZL| ∠
2
I =  V  ; I =

2


Z Z 0 ∠θ [ use complex analysis in maths.]
π
⇔ ZL = ωL ∠ .
=I  V0  π
 ∠ − − θ 2
Z 2
 0  Now we know that V = I Z L

I = I ∠ –  π + θ   … (iii) V0 ∠0
0   = I
2  π
ωL∠
π  2
Phase of current = –  + θ 
 2  I V0 ∠ − π
=
π ωL 2
Phase of voltage = –
2 π
⇒ I = I0 ∠ −
∴ Depending upon the ‘θ’ we can speak more about 2
the relation between fV and φI.
Phase of voltage = ∠ 0 = zero
Sol 5: Let V = V0 sin (wt + θ) be an ac voltage source.
Then
1/2
T 2 
 ∫ V dt 
  V(0)
Vrms =  0 
T
 dt 
 ∫ 
 0  I
1/2
T 2 

2
 ∫ V0 sin (ωt + θ) 

 
-

2
Vrms =  0  π π
 T  Phase of current = ∠ − = −
  2 2
 
Hence current lags behind the voltage by an angle of
now for simplifying the calculation,
π
∴ We put θ = 0, and solve;  .
2
V 
we get Vrms =  0  Sol 8: ω = 2pf
 2
Now as derived in Q.4;
2 3 . 5 0 | Alternating Current

−i −i V0
ZC = = ∴ vrms =
ωC 2πfC
2

Sol 9: V= V0 ∠0 [In phasor] …..(i) V0 = (vrms) 2

C V0 = (220) ( 2)

V0 = 311 V.

V = V0 cos t  And given f = 50 HZ;


ω = 2pf= 2π(50) = 100 p
ω = 314
∴ v = 311 sin (314 t)
−i  1  π
ZC = =  ∠ − (b) Given Vrms = 100 2V;
ωC  ω C  2
V0
 π We know that Vrms =
= Z C | Z C | ∠ θ ; for i → ∠ −  2
 2 
π Comparing both of them;
 −i → ∠− 
 2 V0 = 200 V

I V= V0 ∠0 V = 200 sin (ωt)
Now =
Z  1  π
 ∠ − V = 200 sin (314 t)
 ωC  2

Now; ω =
I = V ωC∠ π T
0
2
 2π t 
⇒ V = 200 sin  
I= I ∠ π …..(ii)  T 
0
2
T/2
Now power dissipated P = V I standard notation get  2πt 
familiar with this ∫ 200 sin 
 T 
 dt
Average = 0
= 127 V.
 T/2
π
P = (V0∠ 0)  I0 ∠ 
 2 ∫ dt
0
π
P = V 0 I0∠ 0 +
2 Sol 14: Let ‘f’ be the required frequency
π
P = V 0 I0∠ ω = 2pf
2
now V = V0cos (2pft)
π
And cos = zero
2 we are given Vrms = 200 V
Hence P = 0. V0
∴ Vrms =
2
Sol 10: Refer to theory.
V0 = 200 2 V

Sol 11: Refer to theory. ⇒ V 200 2∠0 


= ….(i)
ZL = iωL = i(ω) (i)
Sol 12: Refer to theory.
= iw≡ i2pf
π
Sol 13: (a) Instantaneous voltage V = V0 sinωt now V0 is Z L = 2pf ∠
the maximum possible voltage (or amplitude) 2
V
220 V given is the RMS value of voltage now I =
Z
P hysi cs | 23.51

∴ Imax = 5.0 2 A
I = 200 2∠0
π
2πf∠ Let ∴ I = 5 2 sin(100πt)
2
π 1
I = I ∠ − when t = sec
0
2 300
then
200 2
we want I0 = = 0.9 π 3
2πf I = 5 2 sin = 5 2× 2.5 6 A
=
3 2
200 2
∴f= H ≡ 50 Hz
2π(0.9) Z Sol 17: Vrms = 220
V0 = 2 (Vrms)
Sol 15: V0 = Vrms . 2
V0 = 220 2
100 10f ω = 2pf

V = 220 2 cos (2pft)

V = V0 cos t  V= 220 2 ∠ 0
V = V0 cos (t) C


(a) V0 = 100 2
ω = 2π (50) = 100 p
∴ V = 100 2 cos (100 pt) = 100 2 ∠ 0
−i 1 π
ZC = = ∠ − [In phasor notation]
ZR= R = 100 ωC ωC 2
 −i  −i
ZC =  = = –i (318) W  1  π
 ωC  (100π)(10 × 10−6 ) Z C 
= ∠ −
 ω C  2
∴ Resistance of capacitor is |ZC| ≈ 318 W
V V0 ∠0
(b) now Znet = ZR + ZC Now I = =
Z  1  π
C
 ∠ −
Znet = 100 – i (318)  ωC  2
I = V ωC∠ π + 0
0
 −318  2
Znet= (100)2 + (318)2 ∠ tan−1  
 100 
I = V ωC∠ π
Znet = 334 ∠ –72.5° 0
2

I V= 100 2∠0 = 0.42 ∠72.5=0.527 A
= ∴ Phase of current =
π
Z 334∠ − 72.5 2
(c) Pavg= Vrms Irms cos f Phase of voltage = 0
π π
 0.42  ∴ φI– fv= –0=
= (100)   . cos(72.5) = 29.9 cos (72.5) 2 2
 2 
Sol 18: V = 220 2 cos (50 (2π) t)
Pavg = 9 watt
V = 220 2 cos (100 pt)
Sol 16: f = 50 Hz ∴ ω = 2π × 50 = 100 p
=V 220 2∠0
Irms= 5.0 A
2 3 . 5 2 | Alternating Current

R = 100 Sol 20: V = 220 2 cos (2π (50) t)

10 C
220 V  A
50 H
220 V 
50 H
(a) ZR = R = 100 ⇔ ZR = 100 ∠ 0

V = I Z
V = 220 2 cos (100pt)

I V= 220 2∠0
= V = 220 2 ∠ 0  … (i)
Z 100∠0
Now let ‘C’ be the capacitance of the circuit;
=I 2.2 2∠0
−i −i  1  π
ZC = = =  ∠ −  … (ii)
⇒ I = (2.2) 2 cos (100pt) ωC 2πfc  2 πfc  2
ZR = R = 10Ω = 10 ∠ 0  … (iii)
now I0 = (2.2) 2
Now Znet = ZR + ZC
I0 (2.2)( 2)
Irms = = = 2.2 Amp.
2 2 1
Znet= (10 + ZC) = 10 – i
2πfc
(b) Net power over a full cycle
2
 1 
(Vrms )2 (220)2 |=
Znet | (10)2 +  
= = = 484 watt  2πfc 
R 100
 1 
Sol 19: V = 110 2 cos (2π (70) t) −   1 
tan θ =  2πfc  =  − 
 R   2πfcR 
 
1H  
 −1 
θ = tan–1  2πfRC 
 
110 V

10HZ  −1 
∴ Z = (10)2 + (X C )2 ∠ tan−1    … (iv)
 2πfRC 
Now V = 
IZ
V = 110 2 cos (140 pt)= 110 2 ∠ 0 …(i) 
I = V
ZL = iωL = i (140 π) = i (140 π) Z
|ZL| = 440 W 220 2  −1 
=I ∠0 − tan−1  
π 100 + X 2C  2πfRC 
ZL= 440 ∠ …(ii)
2
220 2
 Now I0 =
I V= 110 2∠0
= 100 + X 2C
Z π
440∠ I0 220
2 Irms = =
2 100 + X 2C
1 π  1   π
=I ∠− =   cos  140πt − 
2 2 2 2 2   2 Irms = 2A (Given)
1 220
I0 = = 0.354 Amp. ⇒2=
2 2 100 + X 2C
P hysi cs | 23.53

100 + X 2C =
(110)2 V = – (813 ∠ 0) (40)
V =
−325∠0
XC = 109.5 A
⇔ V = – 325 cos (50 t)
1 −325
Sol 21: Z = ZR + ZL + Z C = 40 + iωL – Vrms = = −230
ωC 2
 1  (b) Inductance:
Z= 40 + i  ωL − 
 ωC 

Now condition for resonance is Imaginary part of


V = − 
IZL( ) = – (8.13 ∠ 0)  50 × 5∠ 2π 
 
Impedance is zero  π
−  2033 ∠  → (x1)
V=
 2 
R = 40 5H 80F
  π 
V=
− 2033cos  50t +  
  2 
 V  (c) Capacitor:

(
V = − I Z c )
  1 π 
V = 230 2 cos(ωt) VC= – ( 8.13∠0 )  ∠ − 
  50 × 80 × 10−6 2 
= 
V 230 2 < 0 → (1) … (i)
 8.13 π
1 VC = −  ∠− 
∴ ωL – =0  50 × 80 × 10 −6 2
ωC
1 1  π
⇒ ω2 = , ωC = VC = − 2033∠ −  → (x2)
LC LC 2
 
1 1 
ω= =  π 
−6 20 × 10−3 ⇔ VC = – 2033cos  50t −  
5 × 80 × 10   2 

1000 Now from equations (x1) and (x2)


= = 50 rad/s
20 we get VL + VC = 0.
w 50 25
f= = = ~
− 8HZ Study more effectively on Resonance conditions.
2π 2π π

Z =40 + i(0) =40 Sol 22: ZL = iωL = i (100 π) (80 × 10–3)

⇔ Z = 40 < 0 →  … (ii) 80 mH 60F


80 mH 60 µF
V
Now from (i) and (ii), I =
Z
230 V 
I 230 =
2∠0 23 2 230 V,50 ~
= ∠0 HZ
40∠0 4 50 Hz

I = 23 2 cos (50t)
= V 230 2 cos(2π(50)t)
4
=V 230 2 cos(100πt)
I = 8.13 cos (50 t)
Now potential drop across =V 230 2 < 0

(a) Resistance: ZL = i (8π)

( )
V = – IR ZL ⇔ 8π <
π
 … (i)
2
2 3 . 5 4 | Alternating Current

−i −i 500i Power transferred to Inductor


ZC = = = −
ωC 100π × 60 × 10 3π
( ) ( I ) = (290 <π)
− 6
= VL
500 π
ZC ⇔ ∠−  … (ii)
3π 2  π
500i  11.6∠  From
From(i)Xand
→ (1)
(ii)and (4)
 2 
= ZL + Z C = 8pi –
Znet


 500  = (290 × 6) ∠
Znet=  8π − i 2
 3π  ⇒ Znet = – 28 i
 3π 
π = 290 × 6 cos  
⇔ Znet = 28 ∠ −  … (iii)  2 
2
V = Zero
Now V = I Znet ⇒ = I
Znet
Similarly zero for the capacitor to.
230 2∠0
⇒ I = Total power absorbed by the circuit is
π
28∠ −
2 P = V( ) ( I ) = ( 230
)
π
2 ∠ 0  11.6 ∠ 
 2 
I 230 2 ∠ π
=
28 2 (
P = 230 2 × 11.6 ∠
π
2
)
I = 6 ∠ π 
P = ( 230 2 × 11.6 ) cos
… (iv) π
2
 2
π
⇒ I = 6 cos  100πt +  P = zero
 2
11.6
I0 = 6 and Irms= = 8.2 amp Sol 23: Explained in the key.
2
Potential drop across;
Sol 24: Initially
(a) Inductor;
XL = 22 at f1= 200 HZ
 π  π
VL= I . ZL =  11.6∠   8π∠  [ω1 = 2π × 200]
 2  2
(XL)A = ω 1L = 22
VL = (11 6 × 8π) <p
VL = 290 ∠ p→ (x1) ⇒ 2π × 200 L = 22  … (i)

VL = 290 cos (100 pt + π) 22


L= = 1.75 × 10–2 H and finally;
400π
VL = 290; VL =
O ( ) O rms
290
= 205 V
2
f2 = 50Hz

ω22 2π (50)
(b) Capacitor
X2 = ω2L  … (ii)
 π   500 π
VC = I( ) ( Z C ) =  11.6∠  
 2   2 π
∠− 
2 
(i) ⇒ x1 =
(1) 2π × 200 × L
(2)
(ii) x 2 2π × 50 × L
11.6 × 500
VC = ∠0 x1
3π =4
x2
VC = 616 ∠ 0 → (x2) x1 22
x=
2 = = 5.5 ohm.
4 4
⇔ VC = 616 cos (100 pt + 0)
616
(VC)O = 616 (VC)rms= =4
2
P hysi cs | 23.55

Sol 25: Resistance of the lamp 90 8kW 8 × 100


= ⇒ Pi = kW
220 V 100 Pi 90
= = 22 ohm.
10
80
Let ‘L’ be the Inductance of the lamp; ⇒ Pi = kW ⇒ Pi= 8.9 kW.
9
XL = ωL = (100 π) L
Znet = 22 + i (100 πL) −i
Sol 28: XL = ωL ; XC =
ωC
 100πL 
Z=
net (22)2 + (100πL)2 ∠ tan−1   Now Now as ω is increased, both XL and XC increase.
 22 


I = V = 250 2∠0 Sol 29: XL = ωL
Znet  100πL  x1 ω1 ω2
(22)2 + (100πL)2 ∠ tan−1   ⇒ x2 = . x1
 22  =
x2 ω2 ω1
250 2  100πL  ⇒ x2= 2x
I = ∠ − tan−1  
484 + (100πL) 2  22 
 −1 
xc =  
250 2 I0  ωC 
I0 = and Irms=
484 + (100πL)2 2 x1 w2 x 
= ⇔ x2 =  1 
x2 w1  2
250
⇒ Irms = Phasor method:-
484 + (100πL)2
Let V = V0 cos (ωt + q1) be the emf of an AC-source,
Put we are given that Irms = 10 A; then can write this is phasor method as,
250 V = |V| ∠ q1⇔ V = V0 ∠ q1
∴ 10 =
484 + (100πL)2 Now for I = I0cos ( ω t + q2)
484 + (100 πL)2 = 625 ⇔ I = I0 ∠ q2
141
100 πL = 141 ⇒ L = Now let Impedance (Z) ;
100π
11.9 ZRe sis tance = R
⇒L= L = 0.04 H.
100π −i
Z capacitor =
ωC
Sol 26: Current drawn in circuit is maximum when the
(i is iota; complex number)
circuit is in Resonance i.e. the Imaginary part of the
circuit is zero. Zinductor = iωL
Now solve this question exactly as solved in Q. 21. Now in a circuit with series RCL;
i
VS NS Znet
= Z=
R Z C + ZL = =
R + i ωL
Sol 27: = ωC
VP NP
 1 
220 NS Znet = R + i  ωL − →  … (i)
=  ωC 
2200 5000
Now let us write this in phasor notation,
NS = 500 turns.
Znet | Znet | ∠ q
=
Output power
n (efficiency) =
Imput power
2
 1 
8 kW | Znet=
| R 2 +  ωL − 
x=  ωC 
Pi
2 3 . 5 6 | Alternating Current

 1  E = (0.1) (0.1) 1 ⇒ E = 10–2 V


 ωL − 
ω C now applying KVL in mesh (i)
θ = tan–1  R
 
  E – i (i) – i1 (2) = 0
 
 1  E = i + 2i1 ... (i)
2
 ωL − 
 1  ωC 
= R 2 +  ωL −
∴ Z net −1
 ∠ tan  In mesh-(ii);
 ωC   R 
 
  ⇒ E – i (i) – 3i2 = 0
Now for a source of emf V = V0 cos (ωt + q1)
⇒ E = i + 3i2 ... (ii)
Img
⇒ i = i1 + i2 ... (iii)
1
From this we get i = A.
 L -
1
C Znet
220

Sol 2: (A) For an L–R circuit,



Re L 
R T (time constant) =  
R 
⇔ V = V0<q1 1
Now energy stored in magnetic field is LI2 and rate of
V V0 ∠θ1 dissipation of energy is I2R. 2
=I =
Z  1 
 ωL −
net

| Znet | ∠ tan  −1 ωC 
Sol 3: (C) At t = 0, inductor is open circuited
 R 
 
  At t = ∞, inductor is short circuited
 1  At t = 0,
 ωL − 
I = V0 –1 
ωC 
∠ q1– tan R 10 V
| Znet |  
 
 
I0 10
For resonance, imaginary part in eq. (i) is zero!

Exercise 2 10
10
=i = 1 amp
10
Sol 1: (B) Emf induced in rod = BLv At t = ∞
A D 10

2  3 10

R
B C

i1 i2 10
i 10V 10V
=i = = 2 amp.
2
1 Rnet 5
3
∴ Difference = (2 – 1) amp= 1 amp.
(1) E (2)
P hysi cs | 23.57

 L  1
Sol 4: (B) T1 (time constant) during build up =   ⇒ i = 0.6   ⇒ i = 0.3 amp
 2R  2
 L 
T2 during decay =   di
Emf across coil = L
 3R  dt
T 3
∴ 1 = di di
T2 2 = i0 (– (–2) e–2t) ⇒ = 2 i0 e–2t
dt dt

di Emf = 2L (0.6) e–2t


Sol 5: (C) Energy stored per unit time = Li
dt
= 2 (2) (4) = 16 J/s. ⇒ E = 6 e–2t ⇒E = 6 e−2ln 2

ln2 −1 1
di E= 6e ⇒E=6× E = 3V
Sol 6: (C) i = 2 amp = 4 amp/s. 2
dt
Applying KVL, Sol 8: (B) i = 5 amp
4V di
= –103 A/S
dt

1
[Since decreasing; –ve sign]

1 5mH

1H A B
3F 15V
di Q di
⇒ 4 – i (1) – L − =0 VA – i(1) + 15 – L = VB
dt C dt
Q di
⇒ 4 – 2 (1) – 1 (4) – =0 VA – VB = i – 15 + L
C dt
⇒Q=–2×3 VA– VB = 5 – 15 + 5 × 10–3 (–10+3)
⇒ Q = 6C.
VA – VB = 5 – 15 – 5 ⇒ VA – VB = –15 V
 −Rt 
Sol 7: (A)
=i i0 1 − e L 
  Sol 9: (C) When ‘i’ is reversed,
 
1 5mH
6V R L
A i B
15V

 di 
VA + i (1) + 15 – L   = VB
 dt 
6v di
i0
= = 0.6 VA – VB = –i – 15 + L
10 dt
 −10t 
=i 0.6  1 − e 5  ⇒ i = 0.6 (1 – e )
–2t
= –5 – 15 + 5 (+10-3) × 103
 
  di
[i is decreasing against the direction of KVL. Hence
Put t = ln 2 dt
= 103].
(
⇒ i 0.6 1 − e
= 2 n 2
) VA – VB = – 5 – 15 + 5
 n 2−1 =i 0.6  1 − 1  VA – VB = –15 V
⇒ i 0.6  1 − e
=  ⇒  
   2
2 3 . 5 8 | Alternating Current

Sol 10: (A) At t = 0, inductor is open circuited, Sol 12: (D) Just before the switch is opened, let us find
the currents,
at t = ∞, it is short circuited
at t = 0, L i2
A B
i1

12 V R1 R2

10 20
l2 V
i=
Rnet
2V
 RR  4×8 8
2V 2 Rnet=  1 2 = = Ω
i=
Rnet
⇒ i1 =
10 + 20  R1 + RL  12 3

12 9
2 1 i= ⇒ i = amp.
i1 =
⇒= amp. 8 2
30 15
3
Finally; at t = ∞ Now just at the instant switch is opened, i would remain
same
L B
A

10 20 12 V R2

R1

2V
9
2V 2 ∴ VR = i R1 = × 4 VR = 18V
i2 = ⇒ i2 = amp 1 2 1
Rnet 20
Now applying KVL;
1 12 + (VB – VA) – 18 = 0 ⇒ VB – VA = 6 V.
i2 = amp.
10
Sol 13: (C) Power factor,
Sol 11: (D) At t = 0, no current flows in the circuit.  
R
cos φ =  
 2 2 
Fuse L = 4H  (X C − XL ) + R 

V=2sin (100t)
S ~

2V
10Ω 0.1H
As time starts, current starts flowing and at t = ∞,
current in the circuit is infinity. C 10 0.1H
Hence at t = 10, i → ∞ so the fuse will get blown
∴ C
[ Infinity is just an unknown number !]
P hysi cs | 23.59


Im I V=
=
220 2
Z 2
z 2  1 
(100) +  100 − 
 100 πC 
(XC-XL)

 R  1 
R  100 − 
100 πC 
1 π –tan–1  100
cos φ = ⇒ θ=  
2 4  
 

| X C − XL | 220
∴ tan φ = ⇒ |XC – XL| = R irms =
R
1  ≡ 2.2
2

(100)2 +  100 − 
XL = ωL = (0.1) (100) ⇒ XL = 10 Ω.  100πC 

⇒ |XC – XL| = R 2
220  1 
= (100)2 +  100 − 
1 2.2  100πC 
= R + XL
ωC
2
1 1  1 
C= ⇒C= ∴ (100) = 100 +  100 −
2 2

ω(R + XL ) 100(20)  100πC 
1 1
C= × 10−3 ⇒ C = 500 µF. ⇒ 100 − =0
2 100πC
 −1 
1 ∴ XC = – 100 ∴ X C = 
Sol 14: (A) ZL = iωL = × 100 π = i 100 W  ωC 
π
1  XC 
H 100 C Now power factor; φ = tan–1  R 
 
 −100  π
φ = tan–1  100  ⇒ φ = −
  4
π 1
Power factor; cos φ = cos   =
 4 2
ZR = 100 W
Sol 15: (D) For 100 V D.C. source, i =1 amp.
−i −i
ZC
= =
ωC 100πC 100
Hence,=
R = 100Ω
1
Znet = ZR + ZL + Z C
Now for AC source of 100 V
i
Znet = 100 + i (100) –
100πC 100 1 100
i=
Znet ⇒ 2 = Z
net
2

2 1 
Znet= (100) +  100 −  ⇒ Znet = 200
 100πC 
Z=
net R 2 + XL2
 1 
 100 − 
−1
tan  100 πC  ∴ R2 + XL2 = (200)2 ⇒ XL2 = (200)2 – (100)2
 100 
  XL = 174 W
 
2 3 . 6 0 | Alternating Current

ω L = 174 tan q2= 3


174
L= ⇒ L = 0.55 H.
100π R1 3 3
∴ =
R2 4
Sol 16: (A) I = I0 + I1 sin ω t
T
(*) Don’t run to catch cos θ.
2
∫I dt

Use tan θ and simplify!
I2rms = O
T=
T ω
Sol 19: (B) i = 2 sin 100 pt + 2 sin (100 pt + 30°)
∫ dt
O
T
It is similar to superimposition of two vectors with an
(I20 I12 2 angle of 30° in between them
∫ + sin ωt + 2I0 I1 sin ωt)dt
= O inet = i0 sin (100 pt + θ)
T

∫ dt i0 = 22 + 22 + 2(2)(2)cos(30°)
O

I12 T i0 =
I20 T + +0 8 + 8 3 ⇒=
i0 2 2 + 3
2 2 I12
Irms = ⇒ Irms
= I20 +
T 2
Phase diagram will be shown as
π π 1
Sol 17: (D) = ωt ;= 100πt ; t = s. o
4 4 400 i2 = 2 sin (100t + 30 )

Sol 18: (D) for LR circuit; ieffective


  /3
R1
cos q1=   = 0.6  
... (i)
 2 2 
 R 1 + XL  i1 = 2 sin 100t

for CR circuit;
Sol 20: (A) We can speak on nature by observing the
 
R2 phase of final Impedance. If the phase of Impedance is
cos q2=   = 0.5  ... (ii)
 2 2  negative then it is capacitive, else it is inductive.
 R2 + XC 
ω 1
Now when L, C, R of two circuits are joined; ∴ ω' = =
2 2 LC
  ∴ ZR = R
R1 + R 2
cos θ =  
 2 2  1 1 L
 (R1 + R 2 ) + (X C − XL )  ZL = i ω L = i . .L=i
2 LC 2 C
Given that cos θ = 1
−i −i L
∴ XC = XL = X ZC = = = −2i
ωC 1 C
.C
X  2 LC
tan q1=  L 
 R1  3i L 3i L
X  ∴ ZL + Z C =
− ; Znet= R −
tan q2=  C 2 C 2 C

 R2 
 −3i L 
tan θ1 XL R 2  R 2  Znet= Z 0 ∠ tan−1  
= . ≡   2R R 
tan θ2 R1 X C  R1   
4 ∴ -ve phase
tan q1=
3 Hence capacitive.
P hysi cs | 23.61

Previous Years’ Questions Sol 4: (C) For the lamp with direct current,
V = IR
π ⇒ R =8Ω and P =80 × 10 =800 W
Sol 1: (A) As the current i leads the emf e by , it is an
R–C circuit. 4
For ac supply
1 2
XC π Erms
tan φ = or tan = ω C \ωCR = 1 =P I 2
= R R
rms
R 4 R Z2
(220)2 × 8
As ω = 100 rad/s ⇒ Z2 =
800
1 –1
The product of C–R should be s ⇒ Z=22Ω
100
⇒ R 2 + ω2L2 =(22)2
2
 1 
Sol 2: (B, C) Z = R 2 + X 2C = R2 +   ⇒ ωL = 420
 ωC  ⇒ L=
0.065 H
In case (b) capacitance C will be more. Therefore,
impedance Z will be less. Hence, current will be more.
∴Option (b) is correct. JEE Advanced/Boards
Further,
Exercise 1
Vc = V 2 – VR2
Sol 1: At t = 0, we can replace the inductor by open
= V 2 – (IR)2 circuit and at t = ∞, the inductor can be short circuited

In case (b), since current I is more. at t = 0,

Therefore, Vc will be less. 6

dI
Sol 3: = 103 A/s
dt 10 4 4
I
A B
1Ω 15 V 5 mH 10
di 10
∴Induced emf across inductance, |e| = L |e| = i1 = = 1 amp.
dt 10
(5 × 10–3) (103) V = 5 V
At t = ∞,
Since, the current is decreasing, the polarity of this emf
6
would be so as to increase the existing current. The
circuit can be redrawn as

10
I = 5A 4 4
A B
1Ω 15 V 5 mH

Now VA – 5 + 15 + 5 = VB 10 10
i2
= = amp
\VA – VB = – 15 V R eff 8
or VB – VA = 15 V i1 1 8
= = = 0.8 amp
i2 10 10
8
2 3 . 6 2 | Alternating Current

L
Sol 2:
RCV
L L
V = IR ⇒ ⇒
RC(IR) R(RC)I 10
Now {RC} = time constant in RC circuit 20

∴ L 
[RC] = [T] and   = time constant in LR circuit 2V
R 
2
I=
∴ L  Rnet
  = [T]
R 
2
I1 =
 L  [T] −1 10 + 20
∴ =  = [I] .
 RCV  [T] [I] 2
I1 = amp.  … (i)
30
Sol 3: Let us calculate the total energy stored in the at t = ∞,
inductor before switch is shifted.

R2
10
20
L
2V
Here the resistor 10 Ω is shorted.
2 2 1
E R1 I2
= = = amp.
Rnet 20 10
1 2
Energy stored in inductor = LI
2 Sol 5: Let us now derive the current in the circuit as a
1 function of time
= L [Iat t = ∞]2
2
R=10 L=5
 E 
It =∞ =  
 R1  i

2 V
1  E 
∴E= L  
2  R1 

LE2
E= at time t = t; current = i amp;
2R12
using KVL;
Now this is the total heat produced in R2.
di di
V – iR – L = 0 ⇒ V – iR = L
dt dt
Sol 4: This is similar to the Questions 1 (Ex. I). 1 di
⇒ dt =
At t = 0; Inductor is open circuited, L V − iR
At t = ∞, Inductor is short circuited. Integrating;
At t = 0; 1
t
di
i

L0∫ dt = ∫
i
V − iR
0
⇒ i = i0 1 − e−Rt/L
( )
P hysi cs | 23.63

At t = 0, i = zero E = 2kLt
At t = ∞, i = i0 = constant E  2kL 
Current in the coil II is = t
R  R 
Now R = 10Ω, L = 5
dq
i=
(
i = i0 1 − e−2t
) dt
t
At t = 1 sec  2KL 
i −2
q= ∫ i dt ⇒q= ∫  R 
 t dt
i = i0 (1 – e–2) ⇒ = (1 − e ) t =0
i0
t
 2KL  t2
 i

 e2 − 1
 = q= .
 R  2
⇒ q=
2KL 2
2R
t( )
 i0  e2 0

KLt2
Sol 6: i = i0 (1 – e–Rt/L) q= C
R
dq
i= ⇒ q = ∫ i dt
dt
Sol 8: Power factor is cos (θ)
 −Rt 
=q ∫ i0  1 − e L  dt Img
 
 
t −RT 
=q i0 ∫ 1 − e L  dt
 XL+XC
t0 
t 
 −Rt  R
⇒ q = i0 t −  − L  e L  R
 
  R  
0 Given that cos θ = 1 ⇒ θ = 0
 −Rt
L 
⇒ q = i0 t + L eL

−  0 + 
∴ |XL + XC| = 0 ⇒ XL = – XC
 R  R 
 XL = ω L
  −Rt  
⇒ q = i0 t − L  1 − e L   XL =
−1
⇒ ωL =
1
 R  ωC ωC
  
1 1
i0L  −Rt  ω = ⇒L=
⇒ q=
i 0t − 1 − e  LC ω2C
R  L 
ω = 2π (50) = 100 p
L 
One time constant ⇒ t =   1 20
R  L= = = 2H.
(100π) C π2 2

L i0L
⇒ q = i0 . −
R R
(
1 − e−1 ) Sol 9: We know that VR and VL will have a phase
π
i0L i0L  1 i0L difference of .
q= − 1 −  ⇒ q = 2
R R  e Re
EL Vnet = VR2 + VC2 = 162 + 122 = 20V.
q=
R 2e
Sol 10: Resistance of Lamp = R
Sol 7: Given mutual inductance between coils = M
 V 2   100 × 100 
And I1 = kt2 R  =
=  =  200Ω
 P   50 
 
dI
∴ EMF induced in second coil= L = L [2kt]
dt Maximum current the lamp can sustain,
2 3 . 6 4 | Alternating Current

Pmax now at t = 0.1 ln 2, S2 is open;


imax =
V

50 1
i=
max = amp. 40
100 2 100 V
Now in the given conditions;
(200 V, 50 HZ ) 10

200V
i= = 1 amp which is greater than 0.5 amp. 1H
200Ω
Hence we need to increase the Impedance by using a
capacitor of capacitance ‘C’. Such that ‘ I’ will be equal

to
1
amp.
∴ inew =
100
50
(
1 − e−50t )
2

∴=
Z
 1 
R +2
2 (
= 2 1 − e−50t 
inew ) … (ii)

 ωC  But this equation; at t’ = 0, we get inew = 0

200 But this is not true; Since there is a current flowing in


I= the circuit at that instant.
2
 1 
R2 +   Also t’ = 0 ⇒ t = 0.1 ln 2 sec.
 100 
∴ t’ = t – 0.1 ln 2
1 1 200
I = amp ⇒ =
2 2 2 ∴ inew = i0 1 − e−50(t −0.1 ln2)  ; t ≥ 0.1 ln 2  ... (iii)
 1   
R2 +  
 ωC  100
2
i0
= = 2 amp.
 1  2
50
R2 +   = (400)
 ω C  using equation (iii) at time t = 0.1 ln 2, i = 0
But this is not true, since there is a current flowing in
2
 1  2 the circuit guided by the equation,
(200)2 +   = (400)
 ω C  i = 10 (1 – e–10t) [from eq.(i)]
ω = 2π (50) = 100 p now at t = 0.1 ln 2
Solving this will give the value of ‘C’.
i = 10 (1 – e–10t(0.1)ln2)
 1
100 
−10t 
=i 10  1 −  ⇒ i = 5 amp.
Sol=
11: i 1 − e e   2
10  
 
∴ inew = 5 + 2 1 − e−50(t −0.1 ln2) 
 
S1
At time t = 0.2 ln2
 ln2−5 
40 S2 inew = 5 + 2 1 − e−50(t −0.1 ln2)  = 5 + 2 1 − e 
100 V  
 1
10 =5 + 2 1 − 
 25 

 31 
1H inew = 5 + 2  
 32 

(
=i 10 1 − e−10t  ) … (i)

inew =  5 +

31 
16
 amp. = 6.94 amp.

P hysi cs | 23.65

Sol 12: After switch is closed; 20


10 30 i3 =
A 11
(i) t = 0; open circuiting the inductor; B
i2
100 10
i1= i2= = amp.
30 3 20
10 i1 30

i2 D C
20
100 V 20 ∴ Hence the current in loop ABCD will be amp.
11
And this current will start decaying to zero
∴ At t = ∞, i = zero.
(ii) now at t = ∞ ;
inductor is short circuited, Sol 13: Applying KVL;
10 i1 30 R i(t)
i2
100 20
E L

100
i1 =
Rnet di
E – i (t) R – L =0
dt
100 50
i1 = = amp di
22 11 i(t) = 3 + 5t ⇒ =5
dt
10 i1 i3 30 E = R i(t) + L(5) ⇒ E = 4(3 + 5t) + 5(6)

i2 E = 42 + 20t

100 20 Sol 14: Now when capacitance is removed;


R L CR L C

and i1= i2 + i3
200 V, ~ 200 V 
2i2
2i2 = 3i3 ⇒ i3= 300 rad/s. 300 rad/s
3
2i2 5i2
i 1 = i2 + ⇒ i1 =
3 3
V = 200 2 cos(300t)
3 3  50  30
i2 = i=  = amp.
5 1 5  11  11 =V 200 2∠0
20
i3 = amp. Znet
= ZR + ZL
11
(iii) Now when switch is open Znet = R + iωL

(a) Immediately after that, current through inductor will  ωL 


be same as just before Znet = R 2 ω2L2 ∠ tan−1  
 R 

I = V
Z net
2 3 . 6 6 | Alternating Current

Sol 15: Maximum current flows when the circuit is in


200 2∠0
I = resonance
P Q
 ωL 
R 2 + ω2L2 ∠ tan−1  
P Q
R
 R  L
L R
200 2  ωL  1µF 32 4.9 H 68
=I ∠ − tan−1   1F 32 4.9 H 68
R 2 + ω2L2  R  ~

Now given that current lags behind voltage by 60°,
 ωL 
∴ tan–1  R  = 60
  =V 10 2 cos(ωt)
ωL =V 10 2∠0
∴ = 3 ⇔ XL = R 3 → (x1)
R 1
i.e. ω =
R 3 100 3 LC
L= ⇒L=
ω 300 1 1
ω= ⇒ ω=
1 1 × 10 −6
× 4.9 × 10 −3
49 × 10−10
L= H.
3 1
ω= × 105 rad/s.
7
Now when the inductance is removed;
By intuition we can say that Impedance of Box P is (32)2 + (X C )2

 XC  π −1 −1
tan–1  R  = XC
= = = –70W
  3 ωC 1 5
× 10 × 10 −6

XC 7
= 3 ⇒ X C = 3 R → (x2)
R
∴ ZP
= (32)2 + (70)2
1 1
=R 3 ⇒ C =
ωC R 3ω | ZP | = 77 ohm,
1 100
C= C
⇒= µF And impedance of coil Q is (68)2 + (XL )2
100. 3 (300) 3 3
1
Now when all together are present XL= ω L = × 105 × 4.9 × 10–3
7
Znet = ZR + ZL + Z C = 100 + iR 3 − iR 3 XL = 70 W
[From X1and X2]
∴ Impedance = (68)2 + (70)2
Znet = 100
| Z Q | = 98 W
Znet
= 100∠0
Znet = 32 – 70 i + 68 + 70 i
I = V 200 2∠0 I 2 2∠0
= ⇒= Znet = 100 W
Z
net
100∠0

=I 10 2 ∠0 ⇒=
I 2
∠0
power = VI
  = (200 2) (2 2)cos(0)
100 10

P = 800 W Voltage across P; VP = (Irms) ( | ZP | )

 200 2  2 2   2
Pavg = Vrms . Irms =      
  10 
 2   2 
  =   . (77)
2
Pavg = 400 W.
VP = 7.7 V
P hysi cs | 23.67

1
Voltage across Q; VQ = (Irms) ( | ZP | ) = (98) 1
10 |XL – XC| = R; ωL − =R
ωC
VQ = 9.8 V.
ω2L − 1
= R ; ω 2L – ωCR – 1 = 0
Sol 16: ωr = 4 × 10 rad/s. 5 ωC

Given Va – Vb = 60 V Solving this would give us

and Vb – VC = 40 V ω = 8 × 105 rad/s.

120
Sol 17: V = 220 2 sin (100 pt)
d
a b c
=V 220 2∠0
 = ZL + ZR = i (100 π × 35 × 10–3) + 11
Znet

Znet = 11 i + 11

We know that during resonance,  11 


Znet= 112 + 112 ∠ tan−1  
VL + VC = 0  11 
∴ π
VC = – 40 V Vc – Vd = – 40V Z
= net 11 2∠
4
(Va – Vb) = irms R 
I V= 220 2∠0 ; I= 20∠ − π
=
1
60 = irms . 120 ⇒ irms = amp. Z π 4
2 11 2∠
4
Now, Vb – VC = (irms) . ZL
 π
⇔ I = 20 sin  100πt − 
40 40  4
40 = (irms) . ( ZL ) ⇒ ZL = = = 80Ω
irms 1
ω L = 80 2
Exercise 2
(4 × 105) L = 80; L = 0.2 mH
Single Correct Choice Type
Now Vc – Vd = – 40
Sol 1: (C) Current is induced by varying magnetic flux.
i.e. irms . Z c = − 40
Here there is no such phenomena as flux linked with
−1 the coil is zero. Hence induced current is zero.
Z c = −80 ; = −80
ωC
1 1 Sol 2: (D)
C= ;C=
80ω 80 × 4 × 105 t
1
C= µF. i1
32
Current i2 is constant and positive i.e. from ‘c’ to ‘d’ have
 | X − XC |
tan θ =  L  i1 has to be from ‘b‘ to ‘a’. Hence negative
 R  di
L
Also i2 = dt
RL

XL+XC di
∴ = constant
cons tant
dt
Hence i1 versus t is as shown.

R di
π Sol 3: (A) Emf induced across inductor = L
Now at θ = dt
4
2 3 . 6 8 | Alternating Current

 Rt  Sol 10: (B) Let Z A be the Impedance of element A, and


i = i0  1 − e − L  ZB be that of element B.
 
 
Initially; when R is connected to A;
di   R  − Rt  di i0R  − L 
Rt
= i0  −  −  e L  ⇒ = e  Znet = R + Z A .
dt   L  dt L  
    Z 

Rt ⇔ Znet= Z 2A + R 2 ∠ tan−1  A 
e = i 0R . e L  … (i)  R 

Rt i = V
i = i 0 – i0 e − L
Z
e V Z 
=i i0 − =i ∠ − tan−1  A
R 
Z 2A + R 2  R 
e
= −i + i0
R Given that current is lagging behind voltage by angle
e = R (–i + i0) [y = –mx + c] ‘q1’
 ZA 
Hence graph A.
∴ tan–1  R  = q1  … (i)
 
di When R is connected to B
Sol 4: (A) Self-induction Emf = –L
dt
Z 
di1 di2 di1 di2 Z= ZB2 + R 2 ∠ tan−1  B 
< ⇒– >–  R 
dt dt dt dt
V Z 
E1> E2. =i ∠ − tan−1  B 
ZB2 +R 2
 R 
L
Sol 5: (A) We know that RC and will have dimensions Given that current leads voltage by ‘q2’
R
1 R  ZB 
of time. Hence and will have dimensions of ∴
RC L q2= –tan–1  R   … (ii)
 
frequency.
Using same method, when R, A, B are connected,
Sol 6: (A) Refer to Questions – 3 (Ex –I JEE Advanced)  Z A + ZB 
θ = –tan–1  R
  … (iii)
 
1 2 tan (– θ) = tan (–q2) + tan q1
Sol 7: (A) LI = 32J  … (i)
2
tan θ = tan q2– tan q1
I2R = 320 … (ii)
(1) L 2 × 32
= = Sol 11: (B) Resonance is a condition of maximum power
(2) R 320
Hence cos φ = 1.
L
τ= = 0.2s.
R
Sol 12: (B) In calculating the rms value, we square each
value.
Sol 8: (B) In an L-R decay circuit, the initial current at
t=0 is 1. The total charge that has inductor has reduced
to onefourth of its initial value is LI/2R

1 2
Sol 9: (C) LI = U
2
A
I2R = P
Hence both A and B have same square value at every
L 2U
T= = point.
R P
P hysi cs | 23.69

2
 1  π
Znet =  ωL −  ∠
 ωC  2

B

i = V ⇒ V0 π
∠−
Z 2 2
Hence irmsA = irmsB net  1 
 ωL − 
 ωC 
Here we have every value greater than that of Irmsin
VL = i Z L
graph A or graph B.
 
 
 V0 π  π
=VL  ∠−  ωL ∠
  2 2  2
  ωL − 1  
  ωC  
 

∴ (irms)C> IA = IB. π π
VL = V1 ∠ −
2 2
Sol 13: (D) Initially in LR circuit; VC = i Z C
 R   R  π π
cos θ1 =   ⇒cos q1=   VC= V1 ∠ − −
 2 2   R 10  2 2
 R + 9R 
1 π π 
P1 = VC= V1 ∠ −  + 
10 2 2 
Now finally Hence phase difference between VL and VC will be π and
XL – XC = 3R – R = 2R π
between VL and I will be ± . Graph D satisfies all the
conditions. 2
 R 
cos θ2 =  
 2 2 
 R + 4R  Sol 15: (A) Let us consider mesh (1);
1
P2 = XC
5
P1 1 P2 i1
⇒ = . 5 ⇒ = 2 (1)
P2 10 P1

i2 (2)
Sol 14: (D) Znet
= ZL + Z C
L C XL R

V V0 < 0
=
1 π
Z=
1 Z=
C ∠−
ωC 2
 i V V0 ∠0
=1 =
V = V0 cos t Z 1 1 π
∠−
 −i  ωC 2
Znet = i ωL +  
 ωC  i1 = V0 ωC∠
π
 ... (i)
2
 1  Now in mesh (2)
⇒ Znet = i  ωL − 
 ωC 
Z 2 = ZR + ZL = R + i ω L
2 3 . 7 0 | Alternating Current

 ωL  B2
Z=
2 R 2 + (ωL)2 ∠ tan−1  
 R 

i V V∠0
=2 = B2
Z 2  ωL 
R 2 + (ωL)2 ∠ tan−1  
 R 
i = i1 ∠ − tan−1  ωL 
2 0  
 R 
At t = ∞;
π  ωL  R
Phase difference between i1 and i2= − tan−1  
i
B2
2  R 
π X 
= − tan−1  L  R
2 R  i
B2

2i
Multiple Correct Choice Type

Sol 16: (D) Using intuition;


Let us go for capacitance in the box Hence B2 lights up early; but finally both B1 and B2 shine
∴ Q = CV with equal brightness.

dq dv
=C Sol 19: (B)
dt dt
dq i1 L i2
Given i = = constant
dt
i
dv
∴ = constant
dt
∴ Graph looks like a straight line.
(1) (2)
dv
i=C
dt Just after switch is closed, Inductor tries to oppose the
8−2 current ‘i1’. Hence i1< i2. As time goes on, the opposition
Slope of the graph = =2 given by inductor reduces.
3
∴ This opposition is due to the induced EMF in ‘L’.
i = 2C = 1 amp
1
C= C = 0.5 C.
2 Sol 20: (B, C, D) Emf induced in coil 1 = L1 di1
dt
L  di
Sol 17: (D) Time constant τ =  E2 = L2 2
R  dt
1 2 di1 di2
Energy stored in magnetic field = LI Given that =
2 dt dt
Power dissipated in resistor = I R 2
E1 L1
∴ = = 4
1 2  E2 L2
 LI 
∴ 22 = τ V2 1
2
 IR  ∴ =
  V1 4
And also given that power given to the two coils is
Sol 18: (A) At t = 0; same,
∴ Vi i1 = V2 i2
P hysi cs | 23.71

i1 V2 i1 1 Assertion Reasoning Type


= ⇒ =
i2 V1 i2 4
Sol 25: (C) Magnetic field is into the page
1 1
W1 = L1 I12 and W2 = L2 I22
2 2
2 2
W1 L   I  W 8 1
=  1   1  ⇒ 1 =    
W2  L2   I2  W2  2   4 
W1 1
∴ = .
W2 4

L
Sol 21: (A, B, C) RC and will have the dimensions
R As resistance is increasing, current decreases
1 R
of time and hence and will have dimensions of
RC L ∴ Magnetic field decreases.
frequency.
Hence there will b e a clockwise current in the ring.
Sol 22: (D) When just after battery is connected, current
Sol 26: (D) In an LCR circuit,
is zero in the circuit, and hence will follow magnetic
1  |Z|= R 2 + (XL − X C )2
field energy  LI2  and power delivered (I2R) is also
zero. 2 
Vmax
imax =
 di 
EMF induced is  L  . Hence there is a finite value. R 2 + (XL − X C )2
 dt 
R . Vmax
(VR)max = ;
Sol 23: (B, D) At time t = 0, capacitor is short circuited, R 2 + (XL − X C )2
Inductor is open circuited. ωL . Vmax
(VL )max =
At t = ∞, capacitor is open circuited, R + (XL − X C )2
2

Inductor is short circuited.


Now (VR)max = Vmax; at resonance condition, (XL – XC = 0),
Hence both the options follow from this.
now for (VL)max; we can set conditions,
diB dφ A
Sol 24: (D) M. = (a) R ?0 and (b) XL = XC;
dt dt
This will lead to (VL)max> Vmax.
∆iB ∆φA
M =
∆t ∆t
Sol 27: (A) When circuit is suddenly switched off, there
∆φA
∴ ∆ iB = will be a change in current, and it will lead to induced
M EMF.
4
∆IB = di
2 |E| = L
dt
∆IB = 2A
Now for large ‘L’, E is also high.
∆φB ∆iA
=M
∆t ∆t
∆φB = 2(1) = 2
But given the values of 4 weber.
Hence options D isn’t true.
2 3 . 7 2 | Alternating Current

Comprehension Type Paragraph 2


L R
Paragraph 1 Sol 30: (D)

Sol 28: (D) Now when S1 is opened and S2 is closed




V = V0 cos t
CV - + CV Znet = R + i ω L
 ωL 
L | Znet
= | R 2 + ω2L2 ;=
Z net | Z net | ∠ tan−1  
 R 
I V V0 ∠0
= =
At t = 0; energy stored is purely in capacitor.In this type Z  ωL 
| Znet | ∠ tan−1  
of circuits, charge and current will be in the form of sin  R 
or cos. Thus oscillatory.
V  ωL 
 1  =I ∠ − tan−1  
q = Q0 cos  t  ; Q0 = CV | Znet |  R 
 LC 
Now potential difference across resistance,
−1
i= Q0 sin ω t
LC VR = i × Z R

Q0 CV C  V  ωL  
=i = = V =  0 ∠ − tan−1    R∠0 
L
LC LC  | Znet |  R  
Hence option D. V0 R  ωL 
VR
= ∠ − tan−1  
| Znet |  R 
 1 
Sol 29: (C) q = Q0 cos  t V0R
 LC  (VR)max = ≡ 4 volts (given) … (i)
R 2 + XL2
dq −Q0  1 
dt
=
LC
sin  t
 LC 
( VL ) = ( i ) ( Z L )
 V X   π
d2q −Q0  1  =  0 ∠ − tan−1  L
= cos  t   ωL∠ 
dt2 LC  LC   | Znet | R    2

 
d2q
= −
1
q, = ( )
VL  V0 XL ∠ π − tan−1  XL  
 2 R
dt2 LC 2
 R + XL
2   
Hence option ‘C’. V0 XL
(VL )max = ≡3V  … (ii)
R 2 + XL2

(i)
(1) R 4
= =
(ii) XL 3
(2)

R 4 3R
∴ = ⇒ XL =  … (iii)
XL 3 4
P hysi cs | 23.73

9R 2 25R 2 Previous Years’ Questions


R 2 + XL2 = R 2 + =
16 16
5R Sol 1: In steady state no current will flow through
R 2 + XL2 =  ... (iv) capacitor. Applying Kirchhoff’s second law in loop 1:
4
In equation (i) 2 i2 2F

V0R V0R
= 4 ; =4
2
R + XL2 5R
1
4 1
i2
V0 = 1 V 2 12 V

you can just start from here if you understand how I i1 - i2 i1


wrote them i1
i1 2
X 
VR = 4∠ − tan−1  L 
R  A 3V 2 B
 X 
⇔ VR = 4 cos  ωt − tan−1  L   3

 R  10
π X 
VL = 3 ∠ − tan−1  L 
2 R  – 2i2 + 2(i1 – i2) + 12 = 0
 π X  \2i1 – 4i2 = – 12
⇔ VL = 3 cos  ωt + − tan−1  L  
 2
 R  or i1 – 2i2 = – 6 …(i)

 X  Applying Kirchhoff’s second law in loop 2:


VL≡ 3 sin  ωt − tan−1 L 
 R  – 12 – 2(i1 – i2) + 3 – 2i1 = 0

Given VR = 2 4i1 – 2i2 = – 9  …(ii)

 X  Solving Equations (i) and (ii), we get


∴ 2 = 4 cos  ωt − tan−1  L  
 i2 = 2.5 A and i1 = – 1A
 R 
Now, VA + 3 – 2i1 = VB
1  X 
= cos  ωt − tan−1  L   or VA – VB = 2i1 – 3
2 
 R 
 XL  π = 2 (–1) – 3 = – 5V
∴ ω t – tan  R –1  = → (X ) PR = (i1 – i2)2 R1 = (– 1 – 2.5)2 (2) = 24.5 W
  3 1
1
(b) In position 2: Circuit is as under
π 3 3
Now VL = 3 sin   ; VL = 3 sin 60° =
3 2
3V 2
3
3 3
Sol 31: (B) Vsource = VL + VR = +2 10
2

4+3 3
Vsource = Steady current in R4:
2
3
i0 = = 0.6 A
3+2
2 3 . 7 4 | Alternating Current

Time when current in R4 is half the steady value Sol 6: After a long time, resistance across an inductor
L –3
(10 × 10 ) becomes zero while resistance across capacitor
t1/2 = τL (In 2) =  n (2) =  n (2) becomes infinite. Hence, net external resistance,
R 5
R
= 1.386 × 10–4 s +R
3R
Rnet = 2 =
1 2 1 2 4
U= Li = (10 × 10–3) (0.3)2 = 4.5 × 104J
2 2 2E
Current through the batteries, i =
3R
+r +r
Sol 2: In circuit (p): I can’t be non-zero in steady state. 4 1 2
Given that potential across the terminals of cell A is
In circuit (q): V1 = 0 and V2 = 2I = V (also) zero.

In circuit (r): V1 = XLI = (2pfL)I ∴E – iri = 0

= (2π × 50 × 6 × 10–3)I = 1.88I  2E 


or E –   r1 = 0
V2 = 2I  3R / 4 + r1 + r2 
4
In circuit (s): V1 = XLI = 1.88 I Solving this equation, we get, R = (r – r )
3 1 2
 1 
V2 = XCI =   Sol 7: Inductive reactance XL = ωL
 2πfC 
= (50) (2π) (35 × 10–3) = 11W
 1 
=  = I = (1061) I
–6 
 2π × 50 × 3 × 10  Impedance Z = R 2 + XL2 = (11)2 + 11)2

In circuit (t): V1 = IR = (1000) I = 11 2 Ω


V2 = XCI = (1061)I Given Vrms = 220 V
Therefore the correct options are as under
Hence, amplitude of valtage V0 = 2 Vrms
(A) → r, s, t (B) → q, r, s, t
= 220 2 V
(C) → q or p, q (D) → q, r, s, t
V0 220 2
∴ Amplitude of current i0 = =
Sol 3: (B) Charge on capacitor at time t is Z 11 2
q = q0 (1 – e ) –t/τ or i0 = 20 A
Here q0 = CV & t = 2t X  –1  11 
Phase difference φ = tan–1  L  = tan  
Here q0 = CV(1 – e–2τ/τ) = CV (1 – e–2) R   11 
π
φ=
4
Sol 4: (B) From conservation of energy,
In L-R circuit voltage leads the current. Hence,
1 2 1 C instantaneous current in the circuit is,
LImax = CV 2 ∴ lmax = V
2 2 L i = (20 A) sin (wt – π/4)
Corresponding i-t graph is shown in figure.
Sol 5: (C) Comparing the LC oscillations with normal
SHM, we get V,I V=
2
dQ
=–wQ 2
20 i=20 sin (t-/4)
dt2 9T/8
O t
1
Here, w = 2
-10 2 T/8 T/2 5T/8
LC

d2Q
\Q = – LC
dt2
P hysi cs | 23.75

Sol 8: (C) When e– has zero kinetic energy total energy


is shared by antineutrino and proton. This time energy
of antineutrino is its maximum possible kinetic energy.
As antineutrino is very light mass in comparison to
proton so it will have almost contribution in total
energy.
∴ Its energy is almost 0.8 × 106 eV

Sol 9: (C, D) As current leads voltage by π / 2 in the


given circuit initially, then ac voltage can be represented
as
=V V0 sin ωt
q CV0 sin=
∴= ωt Q sin ωt
Where, Q= 2 × 10−3 C
3
•• At t = 7 π / 6ω ; I =− I and hence current is
anticlockwise 2 0


•• Current ‘i’ immediately after t = is

Vc + 50
=i = 10 A
R
Charge flow = 2 × 10−6 C
Q final − Q(7 π /6ω) =

Potrebbero piacerti anche